Vous êtes sur la page 1sur 68

Haematology - COAGULATION

Name the main systems involved in preventing and arresting blood loss

Blood coagulation (haemostasis)


• is a host defence mechanism
• prevention of blood loss from intact vessels
• protects the integrity of the vascular system after tissue injury
• is normally quiescent but becomes active within seconds after tissue injury

Coordinated haemostasis requires the interaction of


• Vascular endothelium
• Platelets
• Coagulation proteins
• Leucocytes

and the formation of a haemostatic plug:


1. Vessel constriction
2. Formation of an unstable platelet plug
• platelet adhesion
• platelet aggregation
3. Stabilisation of the plug with fibrin
• blood coagulation
4. Dissolution of clot and vessel repair
• fibrinolysis

Outline the contribution of the vascular system to haemostasis

Composition

Functions
• Provision of basement membrane, collagen, elastin and fibronectin of subendothelial
connective tissue
• The endothelial cell has a role in the haemostatic response:
o Synthesis and expression of tissue factor, prostacyclin, vWF, plasminogen
activator, antithrombin III and thrombomodulin
o Initiates coagulation and participates in the binding of platelets to endothelium
o Regulation of fibrinolysis by synthesis and secretion of t-PA and PAI
o Extracellular matrix (contains heparins) around endothelium promotes AT activity
and heparins also release TFPI
o Function as a link between the inflammatory response and coagulation
o Infusion of TNF alpha to normal humans induces a sustained activation of
coagulation with increased turnover of thrombin evident for 6-12 hrs
o TNF alpha induces TF expression on endothelial cells and increases TM
o Endotoxin reduces TM expression and enhances TF release

1
Outline the contribution of platelets to haemostasis

Origin of the platelet


• Derived from the cytoplasm of bone
marrow megakaryocytes
• Disc shaped, anucleate cells with
relatively complex internal structure
reflecting its specific haemostatic
function
• Normal platelet count is 150-400 x
109/l
o A platelet count of less than
10 leads to severe
spontaneous bleeding
9
o Between 10-40 x 10 /l
bleeding is common
9
o Between 40-100 x 10 /l there
is no spontaneous bleeding
but there is bleeding with
trauma
Platelet granules and their function

Location Compound Function


α granule Platelet factor 4 Neutralizes heparin effect
β-thromboglobulin Promotes fibroblast chemotaxis
Platelet derived growth Mitogen for fibroblast; chemotaxis for neutrophils,
factor fibroblasts and smooth muscles
Adhesion molecule; carrier for VIII protecting it
von Willebrand factor from proteolysis
Thrombospondin Promotes platelet-platelet interaction
Fibronectin Adhesion of platelets and fibroblasts
Dense granule ADP Aggregation of platelet
ATP Source of ATP
Serotonin Vasoconstriction
Calcium Coagulation; platelet function

The contents of both α- and dense granules may be released via a system of surface
connecting tubules, during platelet function.

Circulating proteins
These have procoagulant, anticoagulant and fibrinolytic activities.

2
Platelet reactions and primary haemostatic plug formation

A break in the endothelial lining leads to


• Initial adherence of platelets to exposed connective tissue

Platelet adhesion

Platelet aggregation

• The platelet adhesion is potentiated by von Willebrand’s factor (vWF)


• Collagen and thrombin at the site of injury cause adherent platelets to release granules
including ADP, serotonin, fibrinogen, lysosomal enzymes and heparin-neutralising factor
(PF-4)
• Collagen and thromboxane activate platelet PG synthesis → formation of thromboxane
A2

• Thromboxane A2 potentiates platelet release reactions, platelet aggregation and also has
powerful vasoconstrictive ability
• Released ADP causes platelets to swell and aggregate

3
• Additional platelets are drawn to area of injury continuing growth of haemostatic plug
• Released platelet granule enzymes, ADP and thrombosthenin may all contribute to
consolidation of accumulated platelet plug.
• Prostacyclin, produced by endothelial and smooth muscle cells in the vessel wall adjacent
to the area of damage, is important in limiting the extent of the initial platelet plug.
• This unstable plug is usually sufficient to provide temporary control of bleeding.

Describe the classical and modern theories of blood coagulation. Outline blood
coagulation.

Synthesis of clotting factors, fibrinolytic factors and inhibitors takes place in

1. Liver
2. Endothelial cells
3. Megakaryocytes

Most synthesis is in the liver (e.g. vitamin K-dependent factors and fibrinogen), but some
proteins produced in high local concentration in endothelium (e.g. vWF) and megakaryocyte
(e.g. factor V).

Conventionally the coagulation cascade is divided as follows:

http://www.blooddiseasehopkins.org/coagulation.htm has a nice video making this easier to


grasp.

Intrinsic pathway
• Negatively charged subendothelium activates factor XII which is the beginning of the
cascade.
• In association with calcium and the cofactor VIII, activated factor IX activates factor X on
the membrane surface provided by platelet phospholipid (platelet factor 3)
• Following limited activation, factor XII activates prekallikrein to kallikrein, which in turn
activates factor XII reciprocally and fully
• High molecular weight kiniogen (HMWK) is a nonenzymatic accelerator of these
interactions

Extrinsic pathway
• Tissue factor activates factor VII which in turn activates factor X

Common pathway
• Activated factor X, in association with cofactor on phospholipid surface and calcium,
converts prothrombin to thrombin
• Thrombin converts fibrinogen to fibrin

4
Modern hypothesis of coagulation
• The classical waterfall hypothesis fails to accurately represent what happens in vivo
haemostasis.
o Patients with an inherited deficiency of factor XII, prekallikrein or HMWK have no
clinical bleeding problems but a prolonged APTT
o Factor XI deficiency is not always associated with bleeding and its role therefore
unclear
o Factor VII-tissue factor is known to activate not only factor X but also factor IX
o Tissue factor is a natural constituent of many non-vascular cells and can initiate
blood coagulation so a more central role for the tissue factor-F VII complex has
been suggested
• In the revised coagulation cascade tissue factor-VII and factor X are central to the model
• The model also takes into account the novel feedback inhibition of factor VIIa-tissue
factor produced by tissue factor pathway inhibitor (TFPI)

1) Initiation phase
a. Injury of vessel wall leads to contact between blood and subendothelial cells
b. TF exposed and binds to VIIa or VII which is subsequently converted to VIIa
c. TF-VIIa complex activates IX and X
d. Xa binds to Va on the cell surface
2) Amplification phase
a. Xa/Va complex converts small amounts of Prothrombin into thrombin
b. Generated thrombin activates VIII, V, XI and platelets locally
c. XIa converts IX to IXa
d. Activated platelets bind Va, VIIIa and IXa
3) Propagation phase
a. VIIIa/IXa complex activates X on the surfaces of activated platelets
b. Xa in association with Va converts large amounds of Prothrombin into
thrombin – “thrombin burst”
c. “Thrombin burst” → formation of stable fibrin clot

5
Vitamin K in blood coagulation
• Coagulation factors II, VII, IX and X as well as protein C and protein S are dependent on
vitamin K for their normal function
• They are synthesised in an inactive form that cannot bind Ca2+ - only after post-
translational modification by γ carboxylation of glutamic acid residues can they bind
• γ carboxylation is coupled to conversion of vitamin K hydroquinone to epoxide form
• In vit K deficiency there is no γ carboxylation → noncarboxylated forms of above proteins
released into circulation
• These proteins cannot bind Ca2+ ions and thus cannot bind phospholipid surfaces

Outline the fibrinolytic system and its potential clinical application

Fibrinolysis

• Fibrinolysis (like coagulation) is a normal haemostatic response to vascular injury.


• Deposition of fibrin is accompanied by activation of fibrinolysis
• Fibrinogen and fibrin are substrates for the proteolytic action of plasmin.
• Unlike the highly specific action of thrombin on fibrinogen, which results in the cleavage of
only two pairs of tony fibrinopeptides A and B, plasmin cleaves fibrinogen and fibrin at
multiple sites producing a variety of split (degradation) products.
• Plasmin is normally present in its inactive zymogen form, plasminogen, in blood, urine,
and tissue fluids.
• Major activation of the fibrinolytic system follows the release of the tissue plasminogen
activator (t-PA) from endothelial cells.
• t-PA is a serine protease that binds to fibrin which enhances its capacity to convert
thrombus-bound plasminogen into plasmin
• This fibrin dependence of t-PA action strongly localizes plasmin generation by t-PA to the
fibrin clot.
• Release of t-PA occurs after such stimuli as trauma, exercise or emotional stress.
• Activated protein C stimulates fibrinolysis by destroying plasmin inhibitors of t-PA.
• Therapeutic t-PA and urokinase are produced by recombinant DNA technology.
• The fibrinolytic agent streptokinase is a peptide produced by haemolytic streptococci. It
forms a complex with plasminogen, which converts other plasminogen molecules to
plasmin.
• Plasmin has a wider range
of activity than thrombin,
hydrolysing both arginine
and lysine peptide bonds in
a wider range of substrates.
• Tissue plasminogen
activator is inactivated by
PAI-1.
• Circulating plasmin is
inactivated by potent
inhibitors α2-antiplasmin
and α2-macroglobulin. This
prevents widespread
destruction of fibrinogen
and other coagulation factors.

Natural anticoagulants

1) Antithrombin III (AT-III) is a single-chain glycoprotein (61 kDa) which is synthesised


in the liver and endothelium
a. main physiological inhibitor of activated coagulation serine proteases
b. inactivates thrombin, activated factor X, IX, XI
c. activity greatly accelerated by heparin (2000-fold) therefore sometimes
known as heparin co-factor I

6
2) Heparin co-factor II is a single chain glycoprotein also synthesised in the liver
a. complexes with thrombin in a 1:1 stoichiometric ratio thereby inactivating it
b. it is specific for thrombin with no activity against other serine proteases
c. activity amplified 1000-fold by heparin

On the left is a simplified view of


the coagulation cascade

On the right is the antithrombin


anticoagulant pathway by which
excessive coagulation is
prevented. Antithrombin directly
inhibits the coagulation proteins
such as factor Xa and thrombin

3) Protein C/S pathway


a. Protein C is a vit K dependent protein that inhibits blood coagulation and
stimulates fibrinolysis
b. Protein C is activated by thrombin in the presence of the cofactor
thrombomodulin
c. Activated protein C inhibits coagulation cascade by inactivating factor VIIIa
and Va, reducing the rate of thrombin generation
d. An essential cofactor for protein C is protein S – a single chain glycoprotein
synthesised in the liver and endothelium that is vit K dependent (but not a
serine protease)
e. Thrombomodulin is present in tight association with vascular endothelium
and forms complexes with thrombin in a 1:1 stoichiometric ratio
f. Complexed thrombin activates protein C much faster than free thrombin but
does not clot fibrinogen, activate factors V and VIII or aggregate platelets
g. Thrombomodulin-bound thrombin can still be inhibited by ATIII
h. Activated protein C complexes with protein S and calcium ions on platelets
and at the endothelial surface
i. The inhibitory activity of complexed protein C is greatly amplified

On the left is a simplified view of


the coagulation cascade

On the right is the PC/PS


anticoagulant pathway which
inactivates factors Va and VIIIa

4) Tissue factor pathway inhibitor (TFPI) is emerging as the most important


regulatory mechanism in vivo coagulation
a. synthesised by endothelial cells and circulates in plasma bound to low
density lipoproteins
b. also present in platelets and bound to heparan sulphate at the endothelial
surface

7
c. TFPI inhibits coagulation by binding to factor Xa and TF:VIIa complex and
inhibiting their proteolytic activity

TFPI halts continued direct


generation of factor Xa. Thus,
continued Xa formation early
becomes dependent on ongoing
activation factor X by the IXa-VIIIa
phospholipid complex

Abnormal haemostasis – bleeding disorders

Bleeding disorders Thrombosis


Variable severity May manifest be transient, there may be recurring
clotting. Major cause of CVA and MI. Either arterial or
venous
Acquired defects Genetic defects Acquired defects Genetic defects
• Liver • Rare platelet • Antiphospholipid • Clotting factor
disease abnormalities syndrome inhibitor
• Vitamin K • Clotting factor deficiencies
deficiency deficiencies • APCR/FV
• DIC (haemophilias) Leiden,
• Autoimmune Prothrombin
disease gene mutation
(platelet
destruction
• Trauma

Disorders of haemostasis
• Vascular disorders – scurvy, easy bruising
• Platelet disorders – low number or abnormal function
• Coagulation disorders – factor deficiency
• Mixed/consumption – DIC

Bleeding

Clinical features

Platelet disorders Coagulation factor


disorders
Site of bleeding Skin, mucous membranes Soft tissues, joints, muscles
(epistaxis, gum, vaginal, GI
tract)
Petechiae Yes No
Ecchymoses (“bruises”) Small, superficial Large, deep
Haemarthrosis/muscle Extremely rare Common
bleeding
Bleeding after cuts and Yes No
scratches
Bleeding after surgery or Immediate, usually mild Delayed (1-2 days), often
trauma severe

8
Disorders of platelets
• Decreased number: thrombocytopenia
o ↓ production
o ↓ survival – immune (e.g.
autoimmune thrombocytopenic
purpura – ITP)
o increased utilisation – DIC,
haemolytic anaemia

• Defective platelet function


o acquired – drugs: aspirin
o end stage renal disease
o myeloproliferative disease
o myelodysplastic syndrome
o congenital e.g. thrombasthenia

Acute vs chronic ITP

Features Acute ITP Chronic ITP


Peak age Children (2-6 yrs) Adults
Female:Male 1:1 3:1
Preceding infection Common Rare
Onset of symptoms Abrupt Abrupt – indolent
Platelet count at <20,000 <50,000
presentation
Duration 2 – 6 weeks Long-term
Spontaneous remission Common Uncommon

• Incidence of adult ITP increases with age


• Treatment involves
o if platelet count is 20 – 50,000: nothing if not bleeding, steroids or IVIG if
bleeding
o if platelet count is <20,000: steroids if bleeding, steroids, IVIG and
hospitalisation if bleeding

Clotting factor disorders


• Inherited bleeding disorders
o Haemophilia A and B
o von Willebrand’s disease
o other factor deficiencies
• Acquired bleeding disorders
o Liver disease
o Vitamin K deficiency/warfarin overdose
o DIC

Haemophilia
• Congential deficiency of either factor VIII (Haemophilia A – 1/10,000 males) or IX (B –
1/50,000 males)
• Bleeding – haematoma, joint etc
• Gene of X chromosome (carrier females, males suffer)
• Prolonged PTT but normal PT
• FFP or factor replacement – life long

• Clinical manifestations – indistinguishable between A and B


o haemarthrosis (most common) – fixed joints
o soft tissue haematomas (e.g. muscle)
! muscle atrophy
! shortened tendons
o other sites of bleeding

9
! urinary tract
! CNS, neck (may be life-threatening)
o prolonged bleeding after surgery or dental extractions

von Willebrand disease


• Inheritance – autosomal dominant
• Incidence – 1/10,000
• Clinical features – mucocutaneous bleeding
• Classification
o type 1 – partial quantitative deficiency
o type 2 – qualitative deficiency
o type 3 – total quantitative deficiency
• Diagnostic lab tests:

• Treatment
o Cryoprecipitate
! source of fibrinogen, factor VIII and vWF
! only plasma fraction that consistently contains vWF multimers
o DDAVP (deamino-8-arginine vasopressin)
! ↑ plasma VWF levels by stimulating secretion from endothelium
! duration of response is variable
! not generally used in type 2 diabetes
! dosage 0.3 µg/kg q 12 hr IV
o Factor VIII concentrate (intermediate purity)
! virally inactivated product

Vitamin K deficiency
• Source of vit K
o green vegetables
o intestinal flora
• Required for synthesis
o factors II, VII, IX, X
o protein C, S and Z
• Causes of deficiency
o malnutrition
o biliary obstruction
o malabsorption
o antibiotic therapy
• Treatment
o vitamin K
o FFP

Disseminated Intravascular Coagulation

Activation of both coagulation and fibrinolysis triggered by


• sepsis
• trauma – head injury, fat embolism
• malignancy
• obstetrical complications – amniotic fluid embolism, abruption placentae
• vascular disorders
• reaction to toxin (e.g. snake venom, drugs)
• immunological disorders – severe allergic reaction, transplant rejection

10
Mechanism of DIC:

Pathogenesis of DIC:

Therapeutic approaches to DIC:


• Treatment of underlying disorder
• Anticoagulation with heparin
• Platelet transfusion
• FFP
• Coagulation inhibitor concentrate – APC concentrate

Liver disease

1. Decreased synthesis of II, VII, IX, X, XI and fibrinogen


2. Dietary vit K deficiency (inadequate intake of malabsorption)
3. Dysfibrinogenaemia
4. Enhanced fibrinolysis (decreased α2-antiplasmin)
5. DIC
6. Thrombocytopenia due to hypersplenism

Management for haemostatic defects in liver disease


• Treatment for prolonged PT/PTT
• vit K 10 mg SQ x 3 days
• FFP infusion
• 25-30% of plasma volume (1200-1500ml)
• immediate but temporary effect
• Treatment for low fibrinogen
• cryoprecipitate (1 unit/10kg body weight)
• Treatment for DIC (elevated D-dimer, low factor VIII, thrombocytopenia)
• replacement therapy

11
Haematology - THROMBOSIS

Learning objectives
• Understand the factors determining risk of thrombosis
• Understand the mechanisms of action of antithrombotic therapy
• Be able to balance these two risks
• Treatment of thrombosis - understand the use and actions of anticoagulant therapy
• Prevention of thrombosis - understand the factors increasing risk of thrombosis.
The objective is to achieve the optimum balance between risk of thrombosis and risk of bleeding for each patient.

Why do (some) people get thrombosis?


We still rely on Virchow’s triad:
1. the vessel
2. the blood
3. the flow
although we understand some parts of this better than others.

1. The vessel wall.


The vessel wall normally presents an anticoagulant surface to the blood. This is because it expresses molecules that
participate in the anticoagulant mechanisms:
• Thrombomodulin (helps activate protein C)
• Endothelial protein C receptor (EPCR) - presents PC to TM
• Tissue factor pathway inhibitor (TFPI) - inhibits Tissue factor
• Heparans
• Prostacyclin - inhibits platelet activation
Similarly, it does not normally express Tissue factor, thromboxane and adhesion molecules all or which tend to favour
coagulation.

This pattern is reversed by inflammation. Inflammation is the principal change in the endothelium that increases the
likelihood of thrombosis.

It may result from


! infection
! malignancy
! vasculitis
! trauma

because
! anticoagulant molecules are down regulated
! adhesion molecules upregulated
! TF may be expressed
! prostacyclin production decreased

2. The blood
We know most about the blood, but even here we tend to ignore the roles of platelets and leucocytes about which we
know relatively little. However elevated platelet counts do increase the risk of thrombosis.

Thromophilia – a disorder of the haemopoietic system in which there is a tendency to the occurrence of thrombosis

We usually regard plasma as existing in a balance between pro and anticoagulant forces. In this scheme an increased
risk of thrombosis arises from either:
A reduced level of anticoagulant factors
! Protein C
! Protein S
! Antithrombin
OR
An increased level of procoagulant factors
! Factor VIII
! Prothrombin
! Fibrinogen

12
Additional factors
! Factor V Leiden
! Lupus anticoagulant

Factor V Leiden
! results in a reduced level of anticoagulant factor activity as it causes activated protein C resistance.
! In general the potency of these traits is in inverse proportion to their prevalence.
! Factor V Leiden is common, affecting 2-5% of the white population but is quite mild, increasing thrombotic risk by
a factor of approximately 5.

Lupus anticoagulant
! mechanism is unknown
! a property of some antiphospholipid antibodies
! occurs independently of SLE

3. Flow
Reduced blood flow increases the risk of thrombosis. This may be for a number of reasons; accumulation of activated
coagulation factors and/or favouring platelet adhesion. Reduced flow probably accounts for the increased risk of
thrombosis.

Factors affecting blood flow:


! Immobility – surgery, paraparesis, travel
! Compression – tumour, pregnancy
! Viscosity – polycythaemia, paraprotein
! Congenital – vascular abnormalities

Why are we interested in factors affecting thrombotic risk?


1. Understanding these factors allows us to intervene with anticoagulants to prevent thrombosis at times of high
thrombotic risk or in individuals whose individual risk is high.
2. It helps us decide on the duration of anticoagulation.

13
Anticoagulation
This therapy is used for When monitoring UF heparin:
! treatment of thrombosis
! prevention of thrombosis
APTT patient
! immediate and long term APTT ratio =
APTT normal
For immediate anticoagulant therapy the heparins are used:
! Unfractionated heparin – iv infusion Target 1.5 – 2.5 (0.35-0.7 anti Xa u/ml)
! Low molecular weight heparin – sub cut
! Pentasaccharide – sub cut Monitoring LMWH not usually required
All act by potentiating antithrombin and are used for immediate due to reliable pharmacokinetics except:
effect (e.g. treatment of thrombosis). ! renal failure (CrCl<50)
! Later stages of pregnancy
For long term anticoagulation warfarin is administered ! Specfic conditions/procedures
! given orally ! Monitor using anti-Xa assay
! indirect effect by preventing recycling of vit K ! coagulation screen (PT, APTT,
! levels of II, VII, IX and X fall TT) maybe normal even at
! therefore onset of action is delayed therapeutic levels
Monitoring of warfarin is always essential but is difficult because of
numerous other factors
Monitoring warfarin is standardised by
! dietary vit K
using the International Normalised Ratio
! variable absorption
(INR)
! interaction with other drugs
o protein binding
ISI

( )
o competition/induction of cytochromes PT test
INR =
PT norm
The principal anticoagulants in hospital practice remain heparin and Target 2.0 – 4.5 according to indication
warfarin. Their properties are contrasted below.
ISI = International Sensitivity Index

Heparin Warfarin
Onset Immediate Delayed
Action Cofactor for AT Vitamin K antagonist
Administration Parenteral Oral
Monitoring LMWH – none (or anti Xa) INR
UFH – APTT
Pregnancy Does not cross the placenta. Teratogenic
Risk of osteoporosis
Half life 1-2 hours >24 hours
Reversal LMWH – protamine (partial) Vitamin K
UFH – protamine Factor concentrates

Thus their properties are complementary: heparin is used for immediate effect and warfarin for outpatient control in the
longer term. Their principal side effect is the predictable increase in haemorrhagic risk. Heparin has the additional
problems of Heparin induced thrombocytopenia and osteoporosis.

Management
1. Thrombotic event (e.g. DVT/PE)
• Immediate treatment with LMWH
• Start warfarin at the same time – will take several days to take effect
• Overlap heparin with warfarin for 2 days once therapeutic level reached
• Warfarin continued usually for at least six months.

2. Prophylaxis (e.g. surgery, immobility – long plane flight + risk factor, pregnancy, general medical inpatients)
• Recognise patients at risk and situations posing risk
• Give prophylaxis, usually LMWH, when appropriate e.g. clexane 40mg od sc

14
In both cases we balance haemorrhagic risk of therapy against morbidity and mortality of thrombosis.

Should my patient with antithrombin deficiency be on anticoagulants?


Balancing the risks of coagulation in a non-treated patient with the risks of bleeding in a patient treated with anti-
coagulants has led doctors to believe that the answer is NO.

15
Blood Transfusion

Haemolytic Disease of the Newborn (HDN) and the Use of Anti-D

Formation of red cell antibodies is due to:


1. Blood transfusion
2. Fetal red cells enter a woman’s circulation during pregnancy or at the time of delivery (not
uncommon).

! Only IgG antibodies can cross the placenta.


! If the maternal antibody level is high, it can destroy fetal red cells if they have the
corresponding blood group antigen.
! This causes the fetus to be anaemic and red cell destruction causes a build up of bilirubin
(jaundice). This is Haemolytic Disease of the Newborn.
! The antibody most often responsible for serious HDN is anti-D - hence the reason for
always transfusing Rh negative blood to Rh negative females of childbearing age (ie: <50
years).
! Anti-D is usually due to RhD negative women carrying an RhD positive fetus.

Rh D is the most important cause of severe HDN and is also the only one for which we
have preventative treatment.

Other blood groups can cause HDN eg:


1. Anti-c (another Rh antibody)
2. Anti-K (antibody to the K blood group antigen).

IgG ABO antibodies can also cause HDN but this is usually not severe.

Prevention of anti-D formation

Sensitisation of the mother can be prevented by giving her intra-muscular anti-D


immunoglobulin (anti-D Ig) injections at times when she is at risk of a feto-maternal
haemorrhage

At delivery if the baby is RhD positive. RhD positive (fetal) red cells become coated with anti-
D Ig and are then removed by the reticulo-endothelial system of the spleen, before they can
sensitise the mother to produce anti-D antibodies. For it to be effective, the anti-D Ig must be
given within 72 hours of the sensitising ‘event’. Anti-D immunoglobulin only works if the
mother is not already sensitised.

! 500iu of anti-D is capable of preventing sensitisation from 4 ml of Rh D positive red cells (8


ml of blood) that enter the Rh D negative woman’s circulation.
! A larger anti-D Ig dose may be required when a larger fetal bleed is demonstrated.
! So a Kleihauer test (which tests for fetal haemoglobin-containing red cells) is undertaken
after delivery to check whether the fetal bleed was >4ml or red cells.

Anti-D should be given during pregnancy to RhD negative women, if any of these sensitising
events occur:
1. Therapeutic abortions at any time during pregnancy
2. Spontaneous miscarriages that require medical/surgical evacuation of uterus any time
during pregnancy.
3. Other sensitising events eg: amniocentesis, external version, abdominal trauma and
stillbirths.

Dose: 250 iu anti D if <20 weeks ‘gestation’;


500 iu and Kleihauer test (to see if more needed) if > 20 weeks gestation.

16
Routine Antenatal Prophylaxis
Anti-D Ig given at 28 and 34 weeks gestation has been shown to significantly reduce the
sensitisation, due to ‘silent’ bleeds in the 3rd trimester, but not all hospitals do this yet.

! All pregnant women are tested early in pregnancy for ABO and RhD blood group and
antibodies (eg: anti D).
! This is repeated at 28 weeks gestation, to see if new antibodies have developed during
pregnancy.
! If antibodies present, levels are monitored regularly and the fetus is monitored by scanning,
to look for anaemia.
! Intra-uterine transfusions can be given to the fetus at regular intervals if necessary (but this
is risky).

Adverse Reactions to Transfusion

Administration
Follow written procedures ON WARDS – for checking blood before connecting up to patient.
Administered using a special set which incorporates a filter, to ensure that fibrin strands and
large white cell aggregates are not infused.

No drugs or other infusion solutions should be added to any blood component as


haemolysis or drug interactions may occur.

Monitoring
! Record temperature, pulse and blood pressure repeat within 15 minutes of starting the
transfusion, then hourly after.
! Observed for any signs of a reaction, such as fever, rigors, flushing, urticaria, vomiting,
itching, dyspnoea, headache, pain at or near the transfusion site or circulatory failure.
! If these occur, stop transfusion and consult a haematology doctor
! Generally, keep IV open with crystalloid to maintain BP and renal function.
! If signs and symptoms not due to the blood, transfusion can be restarted.
! If mild, such as urticaria, transfusion can be resumed after the side effects have been
treated, e.g. with antihistamines.

Two main causes of an immediate severe reaction are:


1. Wrong (ABO incompatible) blood
2. Bacterial infection of the blood.

Always
1. Check the patient’s details on their wristband against the ‘compatibility label’ on the blood
bag
2. Check the patient for severe sepsis (temperature etc)
3. Check pack is intact (no tears or pin-holes)
4. Start within 30 minutes of removing packs from Blood Bank fridge

! Blood removed from Blood Bank one unit at a time. No re-use if out for more than 30 mins
! Each red cell unit should be infused in less than 6 hours
! Do not warm units of blood before transfusion (if needed in massive transfusions, do so
through a special blood warmer during the transfusion)
! Examine blood packs for evidence of haemolysis (e.g. purple red cell mass, brown or red
plasma) or clotting.
! If reaction due to bacteria is suspected, broad-spectrum antibiotics should be started and
the transfusion lab alerted.

17
Transfusion Reactions

Immediate Delayed
Immune ! Wrong blood’ ABO incompatible DHTR (red cell antibodies)
! Febrile non-haemolytic (Delayed haemolytic transfusion
! Allergic/ anaphylaxis reaction)
! TRALI
Non-Immune ! Bacterial Infection ! Viral infections + other
! Iron overload

Immediate Transfusion Reactions

1. Immediate haemolytic transfusion reactions due to ABO incompatible blood


! Symptoms: Restlessness, a feeling of oppression, chest pain, vomiting, abdominal or flank
pain and facial flushing are common.
! Haemoglobinuria occurring during the transfusion indicate severe haemolysis. Bacterially
contaminated blood has similar effects.

2. Febrile non-haemolytic transfusion reactions


! Due to white cell antibodies in the patient which can react with white cells in the donor's
blood, causing fever and rigors.
! This is less common since leuco-depletion of all donor blood.

3. Allergic reactions: range from urticaria to anaphylaxis


! Due to a reaction between a foreign protein (e.g. from pollen or milk) present in the donor's
plasma and the corresponding antibody in the recipient.
! Urticaria is usually mild, but if severe the transfusion must be stopped and advice sought.

4. Transfusion related acute lung injury (TRALI)


! Very rare and occurs if the donor's plasma contains potent white cell antibodies
incompatible with the recipient's white cells.
! Transfusion may cause a severe reaction characterised by chills, fever, a dry cough and
breathlessness with cardiac failure.
! TRALI is uncommon.
! Cardiac failure due to volume overload produces similar symptoms and is more common.

5. Non-Immunological - Bacterial complications of transfusion –


! These are relatively rare in the UK, but still more common than viral infections.
! Measures to minimise bacterial infection include:
a. Use of disposable collection sets,
b. Clean techniques,
c. Processing in closed systems.
! Also citrate and the bactericidal powers of blood in addition to cold storage of red cells and
plasma will destroy the vast majority of bacteria that may be introduced at collection.
! Bacterial contamination can be fatal.

Delayed Transfusion Reactions

1. Delayed haemolytic transfusion reactions (DHTR)


Many occur in patients who have antibodies (other than ABO), if specially selected blood is
not given. Usually manifested by fever days to weeks after a transfusion, accompanied by a
falling haemoglobin, and jaundice or haemoglobinuria.

18
2. Iron overload
! Each unit of blood has approximately 200mg of iron.
! Overload is a problem in patients requiring long term transfusions e.g. thalassaemia and
aplastic anaemia.

3. Infectious complications of blood transfusion


! For example viral – hep B, CMV
! Serious adverse events are reported to the SHOT scheme (Serious Hazards of
Transfusion)
! A Patient Information Leaflet can be used to inform patients of the risks and benefits of
transfusion before they agree to have a transfusion.

19
Haematology – HAEMATOLOGY OF SYSTEMIC DISEASE

Mechanism and common causes of a raised ESR


• ESR (mm/hr) measures the speed of settling of the red cell layer of anticoagulated whole
blood when allowed to sediment in a vertical calibrated tube.
• Elevation correlates with the presence of rouleaux in a blood film, and is related to high
protein concentrations eg, fibrinogen, gamma globulin and or acute phase proteins.
• Conditions in which these proteins are increased include infection, malignancy and
autoimmune disorders.
• The test is not diagnostic of a specific disease but serves two main functions.
i. A screening investigation or
ii. a mean to monitor disease activity.
• A very high ESR >100mm/hr is unusual in diseases other than myeloma, active
tuberculosis (TB) and systemic autoimmune disorders.
• ESR can be raised because of
i. severe anaemia (RBCs less packed)
ii. plasma proteins increased e.g. fibrinogen, acute phase proteins,
immunoglobulins
• Normal <20mm/hr
• Increases with age (50% of over 50s are > normal)
• ESR higher in women than men
• ESR is used for:
o adjunct to investigation of vague symptoms
o monitoring clinical course of disease
i. e.g. temporal arteritis or polymyalgia rheumatica
ii. adjunct to detecting relapse in Hodgkin’s disease

Two uncommon causes of cancer-associated anaemia are


1) leucoerythroblastic blood picture
2) acquired haemolytic anaemia (see later)

Leucoerythroblastic anaemia: understand the term and be aware of some conditions,


which may cause this

Leucoerythroblastic anaemia
• Presence in the peripheral blood of nucleated red blood cells and immature myeloid cells
these are normally confined to the marrow.
• There will be a variable degree of anaemia and abnormally shaped red blood cells
(RBCs) (anisopoikilocytosis).
• If bone marrow fibrosis (primary or secondary) is present then teardrop shaped RBCs
may also be seen.
• In adults Leucoerythroblastic anaemia is usually associated with
o 1) bone marrow infiltration by cancer either non-haematopoietic e.g. metastatic
carcinoma, or haematopoietic e.g. myeloma, lymphoma/leukaemia
o 2) myelofibrosis,
o 3) military TB or severe fungal infection
• The explanation is usually established by identifying the associated disease and/or by
bone marrow biopsy.

Common haematological changes in cancer


Red cells
Haematological abnormalities are commonly seen in cancer patients associated with either
the disease process or therapy. Of the disorders associated with the disease process,
anaemia of chronic disease is the most common blood disorder.
• This is generally a mild to moderate normochromic normocytic anaemia
• with impaired ability to utilise reticuloendothelial iron in erythroid production.
• Direct infiltration of the bone marrow by metastatic cancer cells (lung, breast,
prostate) can cause a leucorythroblastic anaemia.

20
Increased rate of red cell destruction (haemolyisis) if sufficiently severe can lead to anaemia.
Common features of haemolytic anaemia of whatever aetiology include
• raised reticulocyte count,
• raised lactate dehydrogenase (LDH) and
• raised unconjugated bilirubin.

Two types of haemolytic anaemia are seen in association with cancer:

1) Microangiopathic haemolytic anaemia (MAHA) due to mechanical


intravascular breakdown of RBCs is relatively common with metastatic
adenocarcinoma, in particular gastric cancer. The features are those of
haemolysis with
• the presence of fragmented RBCs seen in the peripheral blood
smear.
• In more severe cases associated with low grade disseminated
intravascular coagulation (DIC) there may also be
thrombocytopenia, and haemoglobinuria.

2) Autoimmune haemolytic anaemia (features of haemolysis plus


a positive direct antiglobulin test/Coombs test) are seen in
association with tumours of the lymphoid system e.g. non
Hodgkin’s lymphoma and chronic lymphocytic leukaemia.
• reticulocytosis
• raised bilirubin
• raised LDH
• positive DAT

A high haemoglobin in cancer or true polycythaemia (raised red cell mass) can be
• Secondary (raised EPO)
o hepatocellular cancer
o bronchial cancer
o renal cancer
• Polcythaemia vera (PV)
o clonal myoproliferative disorder acquired mutations in JAK2

White cells
A range of abnormalities of white cells can be seen in cancer. The more common include
• an elevated leukocyte count secondary to inflammation and tissue damage caused by
the tumour,
• or less frequently due to production of a tumour elaborated cytokine stimulating bone
marrow cell production.
• Leukopenia can be due to marrow infiltration.

Common haematological changes in infection

Haematological changes in infection and inflammation


There are very marked changes in white cells in response to acute and chronic infection both
increased and decreased cell numbers can be seen in response to infection.
In general acute viral infection can cause elevated neutrophil and lymphocyte counts.
However a lymphocytopenia may also be seen in acute viral infection and in certain chronic
infections such as HIV, TB and brucellosis. Acute bacterial infection usually gives rise to a
neutrophilia. Eosinophilia is seen in parasitic infestation.

Neutrophilia
Can be due to
• corticosteroids
• underlying neoplasia

21
• tissue inflammation (e.g. colitis, pancreatitis)
• myeloproliferative/ leukaemic disorders
• infection

Neutrophilia is found in both localised and systemic infections – acute bacterial, fungal and
certain viral infections. However, those infections that characteristically do not produce a
neutrophilia include
• brucella
• typhoid
• many viral infections

To distinguish reactive from malignant neutrophilia:

Reactive Malignant
• presence bands • presence of myelocytes
• toxic granulation • basophils
• signs of infection/inflammation • splenomegaly
• suggestive of a myeloproliferative
disorder such as chronic myeloid
leukaemia

Eosinophilia
• Parasitic infection
• Allergic diseases e.g. asthma, rheumatoid, polyarteritis, pulmonary eosinophilia
• Neoplasms esp. Hodgkin’s, T-cell non-Hodgkin’s lymphoma
• Idiopathic hypereosinophilic syndrome

Monocytosis
Rare but seen in certain chronic infections and primary haematological disorders
• TB, brucella, typhoid
• Viral: CMV, varicella zoster
• sarcoidosis
• chronic myelomonocytic leukaemia (CMML - a myelodysplastic syndrome)

Neutrophils Eosinophils Basophils Monocytes


Infection Bacterial Parasitic Pox viruses Chronic (TB,
brucella)
Inflammation Auto-immune Allergic
Tissue necrosis (asthma, atopy,
drug reactions)
Neoplasia All types Hodgkin’s
NHL
Myeloproliferative CML CML CMML/MDS

Lymphocytosis
An important question is whether it is secondary (i.e. reactive) or primary?
• Secondary (reactive) – polyclonal response to infection, chronic inflammation,
underlying malignancy
• Primary – monoclonal lymphoid proliferation e.g. CLL, NHL. Is it T or B cell?

Infections that can cause reactive lymphocytosis include


• EBV, CMV, Toxoplasma
• infectious hepatitis, rubella, herpes infections
as well as
• autoimmune disorders
• neoplasia
• sarcoidosis

22
Evaluating lymphocytosis by morphology and “light chain restriction” is useful in distinguishing
the cause and whether the lymphocytosis is monoclonal or polyclonal.

1) Morphology
• reactive/atypical lymphocytes (IM)
• small lymphocytes and smear cells (CLL/NHL)
• lymphoblasts (ALL)

Chronic Lymphocytic Acute Lymphoblastic


Leukaemia Leukaemia

2) Light chain restriction

Common haematological changes in systemic auto-immune disease

Autoimmune diseases in particular systemic disorders such as systemic lupus erythematosis


(SLE) and rheumatoid arthritis (RA), which are associated with a chronic inflammatory
response, may also cause haematological changes.

Anaemia
• SLE and RA may be associated with anaemia the mechanisms include commonly
anaemia of chronic disease and less frequently autoimmune haemolytic anaemia.
Neutropenia
• In SLE it is often an immune mediated phenomenon.
• In rheumatoid arthritis neutropenia is uncommon but in 1% of patients with
longstanding (about 15 years) rheumatoid, Felty’s syndrome, a triad of RA,
splenomegaly and neutropenia may develop.
Thrombocytopenia
• In SLE may be immune mediated, sometimes in association with antiphospholipid
antibody syndrome.
• This syndrome is characterised by; prolonged activated partial thromboplastin time
(APTT), thrombocytopenia, thrombosis, and recurrent fetal loss.

23
Haematology – MYELOPROLIFERATIVE DISORDERS

To understand the different features that distinguish the chronic myeloproliferative


syndromes from those of dysplastic syndromes and leukaemic disorders

These are clonal disorders of haemopoietic stem cells. The abnormal clones replace the
normal polyclonal cells. The neoplastic clones maintain the normal ability for terminal
differentiation, but with loss of the normal feedback control switch. This results in mature but
functional cells.

The diseases are associated with variable increase in reactive polyclonal bone marrow
fibrosis which predominates in myelofibrosis. There is also an increase in the incidence of
terminal acute leukaemic transformation in these disorders.

Leukaemia – proliferation without differentiation


Myelodysplastic disorders – Ineffective proliferation and differentiation
Myeloproliferative disorders – Proliferation and full differentiation

To be familiar with the major chronic myeloproliferative disorders;


• Polycythemia vera (also called primary polycythaemia)
• Essential thrombocythaemia.
• Idiopathic myelofibrosis.
• Chronic myeloid leukaemia (discussed in a separate lecture)

For each of the listed conditions, to be familiar with


• The clinical and haematological features
• The differential diagnosis
• The laboratory and other tests used in diagnosis
• The natural history
• Principles of treatment.

Polycythaemia vera (PV)

• Literally a true increase in many cells in the blood.


• The predominant feature is the increased production in red cells leading to an increase in
red cell volume, increased Hb and haematocrit.
• This occurs independently of the mechanisms that normally regulate erythropoiesis.
• There is also an increase in the plasma volume.
• The increase in white cells, predominantly neutrophils is usually modest and of little
clinical significance.
• The increase in platelet counts may play a role in the clinical picture of the disease.
• Aetiology unknown.

Aetiology
• Annual incidence 2-3/100,000
• Slightly more in males 1.2:1
• Mean age at diagnosis is 60 yrs
• 5% below age of 40 yrs

Presentation
• PV may present with features of increased blood viscosity such as headaches, visual
disturbances, nose bleeds and in extreme cases cardiovascular or cerebrovascular
thrombotic events.
• Patients may be plethoric with engorged fundal vessels and may also have
splenomegaly.
• Patients may also be referred after an incidental finding on a routine blood test. Other
distinctive features are aquagenic pruritus and gout.

24
Investigations Aquagenic pruritus - Intense
• Increased Hb and Hct itching that is the result of brief
• Platelets may be increased and this may be extreme contact with water of any
• White cells normal or moderately increased temperature but that does not
• No circulating immature cells produce visible changes in the
skin, associated with local release
• Increased red cell mass and plasma volume of acetylcholine, mast-cell
• Bone marrow: often increased cellularity mainly degranulation, and increased
affecting erythroid cells but may also be seen in other histamine concentrations.
series
• Serum EPO usually low

Treatment
• Aim to reduce HCT below 0.45 by venesection.
• Cytoreductive therapy (see below) may be needed especially if the platelet count is high.
• Reduce risk of thrombosis: aspirin, keep platelets below 400 x 109/l (see treatment of ET)

Prognosis
• Most survive 10 yrs. 65% 15 yrs
• Thrombosis, leukaemia and myelofibrosis

Essential Thrombocythaemia (ET)


• In ET the predominant cells affected are the megakaryocytes leading to a high platelet
count.
• Common presentations are thrombosis (mainly arterial) but also paradoxically abnormal
haemorrhagic tendency may also occur.
• The disease may also be diagnosed incidentally after a routine blood count.
• Some patients may also present with headaches or TIAs.
• A persistent rise of platelets above 600 x109 is needed to make the diagnosis.

Aetiology
• Incidence 1.5 per 100,000
• Mean age: two peaks – 55 years and minor peak at 30 years
• Females:males equal first peak, but females predominate second peak

Clinical features
• Other than haemorrhage and thrombosis the only other feature may be splenomegaly
which is usually modest.
• In most cases the white cell count and haemoglobin are normal.

Diagnostic tests
• It is important to exclude other causes of reactive thrombocytosis (haemorrhage, iron
deficiency, infections, acute and chronic and occult or overt neoplasia). Normal
inflammatory markers (ESR and CRP) may help to exclude those.
• The bone marrow is diagnostic in many cases. There is an increase in megakaryocyes
with some abnormal form and with clustering.
• A small amount of fibrosis may be present but marked fibrosis may suggest the
alternative diagnosis of myelofibrosis.

Treatment
• Aspirin is often used to try to reduce the likelihood of arterial thrombosis but may increase
the risk of bleeding. The aim of the therapy is to reduce the risk of thrombosis. Therefore
risk stratification is needed for rationalisation of treatment. Recent studies have identified
the need for cytoreductive therapy in the following groups:

1. Absolute platelet count >1500 x109/l.


2. Age >60 years
3. Presence of other prothrombotic risk factors
4. Presence of diabetes or hypertension

25
If treatment is started the aim is to keep the platelets between 200 and 400 x109/l. The
cytoreductive agents are:
• Hydroxycarbamide: a mild cytotoxic agent that reduces production of all cell lines,
especially useful in patients with PV but also used in ET.
• Anagrelide: a non-cytotoxic agent that selectively suppresses platelet production,
useful when other cells are normal or there is anaemia (e.g myelofibrosis).
• Alpha interferon: useful in selected patients (pregnant women or others intolerant of
other treatment).

Prognosis
• Normal life span may not be changed in many patients
• Leukaemic transformation in about 5% after many years
• Myelofibrosis also uncommon, unless there is fibrosis at the beginning

Idiopathic myelofibrosis
• This is a condition in which there is a reactive fibrosis of the bone marrow secondary to a
myeloproliferative process – mainly of megakaryocytes and granulocytic cells.
• There is a resultant myeloid metaplasia (haematopoiesis in extramedullary sites such as
liver and spleen) with splenic enlargement.
• Presentation may be due to increased production of platelets mainly although
granulocytic cells may also be increased, with varying degrees of anaemia and
splenomegaly.

Aetiology
• Incidence 0.5 – 1.5 per 100,000
• M=F
• 7th decade. Less common in younger patients
• Secondary to other haematological disease: progression from PV or ET

Presentation
• Incidental in 30%
• Cytopenias: anaemia or thrombocytopenia
• Thrombocytosis
• Splenomegaly: may be massive
o Budd-Chiari syndrome
• Hepatomegaly
• Hypermetabolic state:
o weight loss
o fatigue and dyspnoea
o night sweats
o hyperuricaemia

• Marrow aspiration may yield a blood tap or a dry tap, and diagnosis needs confirmation
with a bone trephine.
• There are characteristic peripheral blood changes including tear drop poikilocytes and a
leucerythroblastic blood film with increased platelets with some giant forms.
• In late stages, thrombocytopenia may also occur.
• Presentation can sometimes occur with the Budd-Chiari syndrome: portal vein thrombosis
with portal hypertension.

Treatment
Often symptomatic:
• Anaemia with transfusions although this becomes difficult in later stages when the spleen
can be massively enlarged.
• Splenectomy may relieve some of the symptoms and reduce the need for transfusions
but may be hazardous and sometimes followed by transformation to acute leukaemia.
• Cytoreductive therapy may be needed if the platelet count is high. Recently treatment
with thalidomide has shown promising result in a few patients, with an improvement in Hb
and in thrombocytopenia.

26
• In patients below the age of 40, in whom the disease is rare, bone marrow transplantation
is curative and could be considered (experimental).

Prognosis
• Median 3 – 5 years but very variable
• Bad prognostic signs:
o severe anaemia <10g/dL
o thrombocytopenia <100 x 109/l
o massive splenomegaly

Pathogenesis of MPD

• All are clonal disorders of haemopoietic stem cells with dysregulation of one or more
lineages and proliferation with maturation
• Evidence that fibrosis is secondary to abnormal megakaryocytes
• Progenitor hypersensitivity to or independence from several growth factors:
o EPO and erythroid colonies in PV
o thrombopoietin and megakaryocytic colonies in ET

Molecular pathogenesis
• No consistent cytogenetic abnormalities
• Loss of heterozygosity of p9 in 30% of cases of PV and some ET
• Possibly an identical mutation in JAK2 gene (constitutive expression) in:
o most patients with PV
o ~50% patients with ET and slightly less with MF

27
Acute Leukaemia – Clinical Aspects and Therapy

Define the term leukaemia and understand the differences between acute and chronic
leukaemia.

! The leukaemias are types of cancer characterised by proliferation of white blood cells.
! These abnormal cells accumulate in the bone marrow (BM) and usually spill over into the
blood, resulting in a raised white cell count (WCC).
! The malignant cells may also infiltrate other organs such as liver, spleen, lymph nodes,
testes or brain.

To state the main cell type involved in each type of acute leukaemia

There are four main subtypes of leukaemia:

1. Acute myeloid leukaemia (AML)


2. Chronic myeloid leukaemia (CML)
3. Acute lymphoblastic leukaemia (ALL)
4. Chronic lymphocytic leukaemia (CLL)

This classification is based primarily on the white cell type involved.


Acute leukaemias # Cells involved are primitive cells (blast cells)
Chronic leukaemias # It is mature cells which predominate:

Neutrophils in the case of chronic myeloid leukaemia.


Lymphocytes in the case of chronic lymphocytic leukaemia.

Each of the four subtypes has different presenting features and natural history.

Acute Myeloid Leukaemia Acute Lymphoblastic Leukaemia


Typical age of Common Adult Leukaemia (80%) Common Childhood Leukaemia (80%)
presentation Prognosis worse with increasing age Prognosis worse with increasing age
Typical presenting Present with problems of bone marrow failure:
features 1. Anaemia – Pallor, Lethargy, Dyspnoea
2. Infection – Mouth, skin, chest, peri-anal and septicaemia (potentially fatal)
3. Bleeding – Spontaneous bruising, DIC, purpura, mucosal and internal bleeding.
4. Others – Gum hypertrophy, organomegaly, headaches, diplopia. Bone pain in
children. Testicular infiltration in boys with ALL.
Distinguishing Important as prognosis and therapy differ. Uses the following techniques:
between the two 1. Morphology – The appearance of cells. Bone marrow examination for diagnosis
and classification.
2. Immunophenotype – The pattern of protein expression on the cell surface (Cell
surface markers)
3. Cytogenetics – Certain Chromosnal aberrations are associated with each.
4. Other studies – FISH, PCR, Lumbar puncture.
Typical ! FBC – Anaemia and thrombocytopenia
abnormalities in ! WCC – Elevated or reduced
the blood count ! Blood Film – Show presence of blast cells. Diagnostic. Auer rods. AML vs ALL.
Also may show aleukaemic leukaemia.
! Most important – Bone Marrow Biopsy - >20% blast cells indicates AL
Approximate Fatal if Untreated
chance of cure in Treatment highly variable depending in many factors. For example leukaemia type
childhood and and age.
adult acute Childhood ALL/AML – 60%
leukaemia Adult AML <60 years – 40% Adult ALL – 25%
Adult AML >60 years – 15% Child ALL – 85%

In ALL Lymphadenopathy is common


AL constitutes half of all the leukaemias seen in practice.

28
Pathogenesis

! The acute leukaemias are cancers of the blood cells.


! Like all cancer, leukaemia is due to acquired genetic damage, in this instance affecting a
haemopoietic precursor cell.
! The genetic damage disturbs the normal growth, survival and/or differentiation of the
progeny of the stem cell and leads to the accumulation of a population of malignant white
blood cells.
! The acquired genetic damage may occur on a background of inherited genetic damage.
! Individuals with this background of inherited damage are said to have an inherited
predisposition to developing cancer.
! With regard to acute leukaemia (AL) individuals with inherited disorders such as Down’s
Syndrome and Fanconi’s anaemia have an increased risk of developing the disease.

Environmental factors associated with AL are:


1. Ionising radiation
2. Benzene containing compounds
3. Chemotherapy agents

! All these factors are known to damage DNA.


! In AL certain chromosomal translocations are often seen in the malignant cells.
! These generally disrupt genes that are involved in cell growth, survival or differentiation.
! In acute leukaemia the malignant cell is a transformed early haematopoietic cell termed a
blast cell: a myeloblast or lymphoblast.
! In AL these cells continue to proliferate but do not differentiate.
! They infiltrate the bone marrow which leads to BM failure.

Management

! Two important aspects are


1. Supportive treatment - Red blood cell and platelet transfusions, antibiotics, general
medicine.
2. Treatment of the underlying leukaemia – Chemotherapy and BMT

! The management of neutropenic sepsis is a critical clinical skill and responsibility.


! Any patient with severe neutropenia (neutrophils < 0.5 x109/l), for whatever cause, is at
risk of rapidly overwhelming bacterial infection, most commonly from gram-negative
organisms.
! They must be treated quickly with broad spectrum antibiotics.
! A severely neutropenic patient should be nursed in protective isolation with regular
observations.
! Any pyrexia, or unexplained episode of tachycardia or hypotension, may indicate
potentially fatal infection.
! Blood cultures and other investigations are then undertaken and the patient started on
immediate antibiotic treatment
! Most hospitals have a protocol for neutropenic fevers and this will give advice on antibiotics
to be used (commonly piperacillin and gentamicin.)

! Specific treatment of AL involves cycles of intensive combination chemotherapy.


! The role of allogeneic bone marrow transplantation in AL treatment is dependent on
individual assessment of age, risk factors, availability of HLA compatible donor and patient
choice.
! Broadly BMT reduces risk of disease relapse at a cost of treatment associated morbidity
and mortality.

29
What is Cancer?

! Loss of control of proliferation - Alterations in the genetic code, defective apoptosis,


! Failure to differentiate
! Metastasis

Chromosones

! Nearly all human malignancy


contains aberrations in
chromosome count or structure
! Such aberrations are recurrent
and may be directly involved in
the development of cancer

! Translocation
! Occur during cell division
Cross over of chromatids
Breakage and aberrant rejoining
! Exact cause unclear
Toxins/drugs
Radioactivity
! Altered DNA sequence
Creation of new genes
Abnormal regulation of genes
Disruption of genes

! Duplication
Common in leukaemia
Disease hotspots+8 in AML
Possible dosage affect
Extra copies of proto-
oncogenes

Deletion
! Common in leukaemia
! Disease hotspots - deletions of 5 & 7 in AML
! Possible loss of tumour suppressor gene
! Possible loss of DNA repair systems

Also Point mutation

Leukaemogenesis

Multiple genetic hits - At least 2 interacting molecular defects, synergise to give leukaemic
phenotype.
1. Chromosomal translocation
2. Loss or gain of genetic material
3. Localised mutations of DNA

Type 1 abnormalities - Promote proliferation & survival. E.g. FLT3, BCR-ABL


Type 2 abnormalities - Block differentiation. E.g t(15;17). Core Binding factor Proteins.

Tyrosine kinase activation – Common theme in leukaemia. Active in the absence of ligand,
chromosomal translocation, point mutation. BCR-ABL fusion gene - Activated abl tyrosine
kinase
FLT3 – Another type of tyrosine kinase.

30
Proto-oncogene dysregulation – May be leukaemogenic. Not sufficient for acute leukaemia.
Associated with chronic leukaemias.

Transcription Factor Dysregulation - Such as novel Core Binding Factors (CBFs) which are
dysfunctional. Prevents transcription of genes. Noy sufficient alone to cause leukaemia.

Chemotherapy

Normal stem cells - Often quiescent, Checkpoints allow repair of DNA damage.
Leukaemia cells - Continuously dividing, lack of cell cycle checkpoint control.

Cell Cycle Specific Non Cell Cycle Specific


! Inhibit DNA synthesis and repair ! Alkylators: cross link DNA
! Death by apoptosis – Immediate or next cell division ! Dividing and non-dividing cells
! Exposure duration important ! Apoptosis: immediate or next cell division
! Methotrexate: folate antagonist ! Secondary Neoplasia
! Cytarabine: incorporated into DNA
! Thioguanine: incorporated into DNA L-asparaginase: E coli enzyme
! Vincristine: mitotic spindle poison ! Hydrolyses serum asparagine
! Deprives leukaemia cells of L-asparagine
! Reduced protein synthesis

Chemotherapy’s toxicity is predictable and effects rapidly dividing cells such as blood, gut and
hair follicles. Some have less predictable reactions and will have side effects such as
Vincristine effects the peripheral nerves.

Combination chemotherapy - Different mechanisms of action, Synergy and non-overlapping


toxicity.

Acute Myeloid Leukaemia Acute Lymphoblastic Leukaemia



! Mainly cell cycle specific drugs ! Mainly cell cycle specific drugs
! 4-5 courses ! 2-3 years of therapy
! Remission induction x2 ! Remission induction
! Consolidation x2-3 ! CNS directed therapy – Intrathecal,
! Around 6 months of therapy systemic, radiation.
! Maintenance therapy

Supportative Care

Blood products

Red cells Platelets Clotting Support Granulocytes


! Keep Hb >8 or 9. ! Keep platelets >10 if well ! Vitamin K (nutritional ! Buffy coat
! Antibody screen ! Keep platelets >20 if febrile or unwell status) ! Not very
every 3 days ! More for bleeding or procedures ! DIC (sepsis) – FFP or effective
! Tranexamic acid cryoprecipitate ! Not routinely
! No NSAIDS/ aspirin/ IM injections used

Infection

Significant cause of morbidity & mortality


Severe overwhelming sepsis
Prevention – Diet, patient hygiene, barrier nursing, antibiotic prophylaxis, daily inspection
Tx - 4 hourly observation, immediate empirical therapy for fever, hypothermia or hypotension.
! 1st line: dual agent gram negative cover - Extended spectrum penicillin, aminoglycoside
! 2nd line: skin organisms
! 3rd line: atypical organisms

31
Fungus Infection
! Prolonged neutropenia, Prolonged steroids, Contaminated food, Building work.
! Example infections – Candida, Aspergillus, mucormycosis
! Fungus prevention - Fresh food, face mask, building work, air filtration, prophylaxis
! Diagnosis – Difficult, blood culture, tissue culture: sterile site, histology, radiology, antigen
testing, PCR.
! Fungus treatment - Medical therapy, surgery

Bone Marrow Transplant

Autologous transplant
1. Stem cell collection
2. Stem cell storage
3. High dose chemotherapy
4. Stem cell re-infusion
Problems with graft contamination.

Allogeneic transplant
! Donor selection & screening
! High dose chemotherapy +/- radiation
! Stem cell re-infusion
Problems with graft versus host disease – Skin, Gut, Liver
High treatment related mortality – Neecs careful patient selection.
GvHD: reduced relapse rate - Donor lymphocyte infusion, reduced intensity conditioning.

32
Chronic Myeloid Leukaemia and Chronic Lymphocytic Leukaemia

The chronic leukaemias are distinguished from acute leukaemias by the presence in the
blood of mature cells and not primitive (blast) cells. In chronic myeloid leukaemia there are
circulating cells of the neutrophil series and in chronic lymphoid leukaemia there are
circulating mature lymphocytes.

Chronic Myeloid Leukaemia

! CML is a disorder predominately of the middle aged (peak incidence 25-45 years).
! In CML there is a malignant population of white blood cells.
! The molecular basis of CML relates to the Philadelphia (Ph) chromosome.
! This is a chromosome 22 with a short long arm which results from a translocation of
material between chromosomes 9 and 22.
! This translocation, t(9;22), is seen in 90-95% of all cases.
! It is an acquired translocation present only in the malignant cells.
! The translocation disrupts two separate genes BCR on chromosome 22 and ABL a
Tyrosine Kinase on chromosome 9 to create a new BCR-ABL fusion gene.
! This new gene results in increased cell proliferation.
! In the chronic phase of CML the malignant cells proliferate excessively, however unlike
AL they retain the ability to differentiate into mature cells which leads to different clinical
and haematological features.

State the Clinical Features of CML


! M:F 1.4:1
! 40-60 years
! Presenting symptoms may relate to hypermetabolism (weight loss, night sweats).
! Splenomegaly, which may be massive is almost always seen.
! Features of anaemia
! Bruising/bleeding
! Gout

Describe the Typical Peripheral Blood Picture


! The disorder may be diagnosed on a routine blood count.
! A FBC shows a high WCC 50-200x109/l.
! The film shows a large number of mature WCC such as neutrophils and myelocytes.
! Basophils may also be increased.
! The Hb and platelet count are usually normal.
! Serum urate may be increased.
! Low neutrophil alkaline phosphatase score
! Normochromic normocytic anaemia
! Serum vitamin B12 $

State the Diagnostic Cytogenetic Abnormality


The Ph chromosome is present in BM cells and molecular studies will reveal the novel BCR-
ABL fusion gene.

Describe the natural history of CML


! The disorder has a chronic phase (CP) of median 3-4 years duration, where the disease
can be controlled by simple treatment.
! It then undergoes blastic transformation to AL, most commonly AML.
! Blast transformation to AL is generally poorly responsive to treatment, unlike the chronic
phase and survival is short once transformation has occurred.

Treatment options during the chronic phase include:


1. Oral hydroxyurea (ribonucleotide reductase inhibitor) to suppress the WCC.
2. Alpha-Interferon may also suppress the WCC and unlike hydroxyurea in some patients
prolongs the duration of chronic phase. It has side effects and requires regular
subcutaneous injections.

33
3. Imatinib mesylate is a novel oral drug which specifically inhibits ABL tyrosine kinase
activity. The agent is extremely effective at controlling the proliferation of chronic phase
cells in CML. It is as yet unclear if it offers a survival advantage over Interferon.
4. Allogeneic bone marrow transplantation in chronic phase is potentially curative, however
this is only an option for patients <55, with an HLA donor. BMT does carry a morbidity
and mortality. Prognosis depends on Age, CMV positivity and HLA match.
5. Splenic Irradiation

Know the Average Survival Duration


Prognosis - Good response to treatment in the chronic phase.
The median survival is 3-5 years, with 20% of patients surviving 10 years or more.

Accelerated Phase and Acute Transformation


! May occur rapidly over days or weeks
! Patients become refractory to therapy
! Accompanied by anaemia, thrombocytopenia and an increase in basophils or blasts in
the peripheral blood
! Enlarged spleen
! Fibrotic bone marrow
! New chromosome abnormalities may be present
! 1/3 cases show lymphoblastic transformation, 2/3 show transformation to AML

Variants of CML
1. Ph negative CML - Molecular changes of CML but no Ph chromosome. Haematological
differences from CML, often with features of myelodysplasia/
2. Juvenile CML

Chronic Lymphocytic Leukaemia

! This is the commonest leukaemia in the western world.


! Proliferation of mature B-lymphocytes
! Occurs in the elderly. Rare before 40 years
! 67% of cases are in those over the age of 65.
! The male:female is 2:1.
! The cause of CLL is not known.

State the Clinical Features of CLL


! More advanced cases are associated with lymphadenopathy, and/or hepatosplenomegaly
and/or anaemia and/or thrombocytopenia.
! Autoimmune disease, particularly autoimmune haemolytic anaemia, may be a feature at
presentation or during the course of the disease.
! Bacterial infections are common in advanced disease and may be related to
immuneparesis.

Describe the Typical Peripheral Blood Picture


! CLL is commonly an incidental finding on a full blood count.
! Typically the white (lymphocyte) count is raised with normal haemoglobin and platelets.
! Smear cells
! Normocytic normochromic anaemia
! Thrombocytopenia
! Bone marrow - Lymphocytic replacement of normal marrow elements
! The bone marrow is always involved and the pattern of involvement provides some
prognostic information.
! Serum immunoglobulins may be suppressed (immuneparesis), there may be a
paraprotein and the direct Coombs test may be positive.

State the Major Prognostic Factors


Prognostic factors include:

34
1. Immunoglobulin gene mutational status (immunophenotyping) - Mutated=good
prognosis and unmutated=bad.
2. Cytogenetics - 13q14 deletion is associated with good prognosis and 11q23 and/or
17p deletion are associated with bad prognosis.

Two clinical staging systems (Rai and Binet) based on these criteria are used to guide
treatment.
Clinical Staging

Binet Staging System

Stage Features % of Patients


A <3 Lymphoid areas 60
B >3 Lymphoid areas 30
C Hb <10g/dl 10
Platelets <100x109/l

Conventional not to treat Stage A

Describe the Natural History of CLL


! Most patients present with early stage disease (lymphocytosis only) and do not require
treatment. The lymphocyte count may remain stable or may increase over a variable time
that may extend over many years.
! If the lymphocyte count rises rapidly or major lymphadenopathy, hepatosplenomegaly,
anaemia or thrombocytopenia develop treatment is instituted.
! A minority (10%) of patients may transform into a high grade Non Hodgkin Lymphoma
(Richter transformation.)
! This presents with rapid progression of lymph node enlargement along with symptoms
such as fever night sweats and weight loss.

! Conventional first line treatment is with an alkylating agent i.e. chlorambucil.


! However, nowadays, the nucleoside analogue fludarabine is increasingly employed
(either alone or in combination with cyclophosphamide) as first line treatment for the
younger patient.

Lines of Treatment

First Line Second Line Refractory


Steroids Purine analogues - Fludarabine High dose methyl
Alkylating agents - Chlorambucil, Combination - Fludarabine+ prednisolone
cyclophosphamide cyclophosphamide and BMT Anti-CD52 - Campath
Purine analogues - Fludarabine

Know the Average Survival Duration


Prognosis - Patients with mutated immunoglobulin genes have a median survival of 25 years
and those with unmutated immunoglobulin genes have a median survival of 8 years.

! Such treatment may produce long-term remissions but is not curative and the disease is
likely to relapse.
! Relapsed disease may be treated with a fludarabine or fludarabine combination.
! The monoclonal antibody Campath has a role in the treatment of fludarabine refractory
disease or disease associated with p53 dysfunction (for example, loss of the p53 gene
i.e. 17p deletion).
! The place of autologous stem cell transplantation is being determined in clinical trials.
! Allogeneic stem cell transplantation is potentially curative but can only be considered in a
minority of patients.

Complications of CLL
Autoimmune haemolytic anaemia 10-15% of patients Positive DAT
Autoimmune thrombocytopenia 5% of patients
Other autoimmune disease

35
THE LYMPHOMAS: HODGKIN AND NON HODGKIN
Dr Donald MacDonald
LEARNING OBJECTIVES
1. To be able to define the term ‘lymphoma’
2. To be able to explain how the lymphomas are classified
3. To state how lymphomas may spread to other parts of the body and to describe the staging of
lymphoma
4. To state how lymphomas usually present, and the general symptoms that patients may have at
diagnosis
5. To know what features at diagnosis of Hodgkin lymphoma and non-Hodgkin’s lymphoma (NHL)
predict a less good outcome with therapy
6. To be able to describe, in outline, how patients with lymphoma may be treated and the major
complications of therapy

The Lymphomas
• Definition: The term ‘lymphoma’ means a neoplastic (malignant) tumour of lymphoid cells.
Lymphomas usually arise in lymph nodes but may arise in other lymphoid organs such as the
spleen or the gut-associated lymphoid tissue.
• Classification: The lymphomas are classified into Hodgkin lymphoma (HL) and the non-Hodgkin’s
lymphomas (NHL). NHL are further subdivided on the basis of cell of origin (B-lymphocyte or T-
lymphocyte) and aggressiveness of disease i.e. indolent (also called low-grade), aggressive (or
intermediate-grade) and very aggressive (or high-grade).
• Incidence There are approximately 200 new cases per year for every million of the population
(around 10,000 new cases a year in the UK). Of these 80% are NHL, 20% HL.

Hodgkin lymphoma
• Clinical features
o HL is more common in males than females.
o The peak incidence is in young adults (15-40 years).
o The aetiology of HL is unknown, but the Epstein-Barr virus (EBV) may be involved in some
cases.
o Usual presentation is with painless enlargement of lymph node/nodes.
o Constitutional symptoms such as fever, night sweats and pruritis may be present. (see
‘Staging’)
o A cyclical fever lasting 1-2 weeks and then subsiding for a similar period is classical but
affects only a minority of patients (Pel-Ebstein fever).
o Alcohol induced lymph node pain
o Mediastinal lymphadenopathy
• The diagnosis is made by biopsy of an affected lymph node. The pathological ‘hallmark’ of HL is the
Reed-Sternberg cell, a binucleate or multinucleate cell, set in a background of lymphocytes and
other reactive cells, the characteristics of which defines the sub-type. These sub-types are termed
lymphocyte-rich (LR), mixed cellularity (MC), nodular sclerosing (NS) or lymphocyte-depleted
(LD). These four entities are termed ‘classical’ HL.
• LR - 5% MC - 17% NS - 78% LD - <1%
• Also recognised is Nodular Lymphocyte Predominant HL, which has different characteristics from
classical HL.
• Staging:
o Following pathological diagnosis of a lymph node biopsy patients are ‘staged’ as to the
extent of their disease, since this has prognostic significance and also may determine the
best approach for therapy.
o Lymphomas can spread to other parts of the body in 3 ways:
! via the lymphatic system,
! via blood vessels,
! or by local spread.

36
o The most useful staging procedure nowadays is a CT scan of thorax and abdomen/pelvis.
o This may be supplemented by bone marrow biopsy, liver biopsy, and other imaging
investigations in selected patients. The staging system (for both HL and NHL) is as follows:
! Stage I: one lymph node region or single extranodal organ / site.
! Stage II: involvement of two or more lymph node regions on the same side of the
diaphragm.
! Stage III: involvement of lymph node regions on both sides of diaphragm
! Stage IV: diffuse or disseminated involvement of extranodal sites (e.g. liver or bone
marrow), with or without lymph node involvement. .
o For the purposes of staging, the spleen is treated as a lymph node. The suffixes ‘A’ or ‘B’ are
used to denote the absence or presence respectively of systemic symptoms i.e. fevers, night
sweats, unexplained weight loss of at least 10% body weight in previous 6 months. The
presence of any of these is denoted by ‘B’ e.g. Stage IIIB
• Outline of therapy:
o Therapy can include radiotherapy, chemotherapy, or both.
o Chemotherapy is often given as a combination of drugs that affect the malignant cells in
different ways, and therefore hopefully have additive or synergistic effects.
! Examples of chemotherapy drugs used in the treatment of HD are anthracyclines
(e.g. doxorubicin), bleomycin, and corticosteroids (e.g.
prednisolone).
o Previously patients with limited disease (Stage IA or IIA) were treated with
radiotherapy alone and had a good chance of cure, but nowadays a short
course of chemotherapy is often given additionally to further improve
outcome.
o Treatment of more advanced disease is usually with longer courses of
chemotherapy; radiotherapy may also be given to areas of bulk disease.
o Relapsed patients may benefit from very intensive therapy including
autologous haematopoietic stem cell transplantation.
! High dose therapy with peripheral blood stem cell support:
! ‘LACE’
• CCNU (lomustine) 200 mg/m2
• Ara-C 4 g/m2
• Cyclophosphamide 5.4 g/m2
• Etoposide 1 g/m2
o Regimes
! MOPP
• Mustine 6 mg/m2 i.v. D1 + D8
• Vincristine (max. 2mg) 1.4 mg/m2 i.v. D1 + D8
• Procarbazine 100 mg/m2 p.o. D1-D14
• Prednisolone 40 mg/m2 p.o. D1-D14
! ABVD
• Adriamycin 25 mg/m2 i.v. D1 + D15
• Bleomycin 10 mg/m2 i.v. D1 + D15
• Vinblastine (max 10mg) 6 mg/m2 i.v. D1 + D15
• DTIC 375 mg/m2 i.v. D1 + D15
o MOPP/ABVD are given as alternating courses at 4-weekly intervals.
ABVD, if used alone, is given at 4-weekly intervals.
! ChlVPP (substitute for MOPP)
• Chlorambucil 6 mg/m2 p.o. D1-D14
• Vinblastine (max 10mg) 6 mg/m2 i.v. D1 + D8
• Procarbazine 100 mg/m2 p.o. D1-D14
• Prednisolone 40 mg/m2 p.o. D1-D14
o Complications of therapy:

37
!Most of the treatments available for HL and other lymphomas carry a risk of
potentially serious complications. Patients with lymphoma have impaired immune
systems and are therefore at risk of infection, including opportunistic infections. This
tendency is exacerbated by the therapy of the disease i.e. chemotherapy and
radiotherapy.
• Bacterial (gram +ve/-ve)
• Viral (e.g. CMV, herpes, hepatitis, influenza)
• Fungal (e.g. candida, aspergillus, pneumocystis)
• Protozoan (e.g. toxoplasmosis)
! Non infection complications
• The majority of therapies available for lymphomas carry with them potentially
serious complications apart from infection. Some of these will inevitably
reduce life expectancy in patients who are otherwise ‘cured’ of their
lymphoma.
o Short-term: hair loss, nausea/vomiting/diarrhoea, mouth soreness,
pancytopenia
o Long-term: permanent damage to vital organs, secondary cancers,
infertility, heart disease, lung disease, other cancers (e.g. lung, breast,
thyroid) including myelodysplasia/acute leukaemia
o Summary:
! Chemo for all
! Benefit of radiotherapy uncertain – reduced relapse, increased complications, risk of
2° cancer.
+ve
3-4 cycles Radiotherapy
Stage I-II
ABVD PET
Stage III-IV Chemotherapy scan
or -ve
Stage II+ LMM 6-8 cycles No further
Treatment
(avoid RT)

• Prognosis: this depends principally on stage. Cure rates ranges from 50-90%. Over 80% of
patients with stage I or II disease are cured compared to only 50% of stage IV patients. Older
patients generally do less well as do those with lymphocyte-depleted histology. 10% die from
disease and a further 10% die from a complication of treatment.

Non-Hodgkin’s Lymphoma (NHL)


• Incidence: NHL accounts for approximately 5% of all cancers. The incidence increases with age.
• Clinical Presentation:
o As in HL, the usual presentation is with painless lymphadenopathy.
o Enlargement of liver and spleen and systemic symptoms may be present.
o Extra-nodal involvement, e.g. of the central nervous system or gastro-intestinal tract, is more
common in NHL than HL.
• Pathological diagnosis:
o Classifies NHL into a variety of types.
o Most of them arise from B cells.
o They are divided according to the histological appearance into
! indolent (also called low-grade),

38
• well differentiated
• smaller cells
• e.g. follicular lymphoma
! aggressive (or intermediate-grade)
! very aggressive (or high-grade).
• Larger cells
• e.g. large B-cell lymphoma
o Paradoxically aggressive lymphomas are more easily cured than indolent lymphomas.
Median survival Response
without Trx to Trx

Burkitt’s lymphoma
T-lymphoblastic Very aggressive Weeks 2-5
Curable
Months 3-12
Diffuse Large B cell Aggressive
Follicular Incurable
Marginal Zone/Malt Years 10-15
Indolent
Mantle Cell???
Small lymphocytic

• Staging is carried out as in HL.


• Outline of Therapy:
o Indolent (low-grade):
! Limited disease (Stage IA) is treated with radiotherapy and is curable.
! More advanced disease is not curable except with extremely intensive therapy in
younger patients.
! Thus the majority of patients with more than limited disease need not be treated
(‘watch and wait’ policy) until disabling symptoms or complications occur, at which
time simple (single agent) oral chemotherapy (e.g. chlorambucil) is used.
• When to treat – ‘B’ symptoms, haemopoietic impairment, sign of progression,
bulky disease
! Newer drugs (e.g. fludarabine, rituximab) and/or more intensive combination
chemotherapy are reserved for relapsing/progressive disease.
• combination drug regimens e.g. CVP, CHOP
• interferon-alpha
• fludarabine (purine analogue)
• rituximab (chimaeric anti-CD20 monoclonal antibody)
o Aggressive and very aggressive (intermediate-grade and high-grade)
! Often curable with intensive combination chemotherapy using a mixture of drugs
usually including an anthracycline (e.g. doxorubicin).
• Combination drug regimens e.g. CHOP
o Cyclophosphamide 750 mg/m2 i.v. D1
o Adriamycin 50 mg/m2 i.v. D1
o Vincristine (max 2 mg) 1.4 mg/m2 i.v. D1
o Prednisolone 40 mg/m2 p.o. D1-D5
! Some patients, particularly those who have relapsed after first-line therapy, benefit
from high-dose therapy with autologous haematopoietic stem cell transplantation.
! A recent advance has been the addition of rituximab (anti-CD20) to the
chemotherapy of B cell NHL.
o As with HL, treatment is associated with risks of infection and other complications.
• Prognosis:
o Median survival 8 years (80-85% with stage III or IV disease)

39
o If patients progress after complete or partial remission they have median survival of 5 years
o Patients progressing within 1 year of remission have 2.5 years median survival
o The cure rate for NHL with therapy ranges from 25-75%, dependent on a number of
prognostic factors (known as IPI):
o Histology +1
o age (adverse over 60y) +1
o serum lactate dehydrogenase (adverse if raised above normal) +1
! marker of rapid cell turnover
o number of extranodal sites involved (adverse if more than one) +1
o effect of disease on normal daily activity +1
! (performance status: adverse if significantly reduced)
o stage of disease (adverse if Stage III or IV) +1

5 year predicted survival by number of risk factors:


• 0-1: 73%
• 2: 51%
• 3: 43%
• 4-5: 26%

40
MYELODYSPLASTIC SYNDROMES
Prof Inderjeet Dokal
Aims of Lecture
1. To provide an understanding of the definition, pathophysiology, clinical features and treatment of
myelodysplasia.
2. To provide an understanding of the definition, pathophysiology, clinical features and treatment of
aplastic anaemia.

• The myelodysplastic syndromes (MDS) are a heterogeneous group of haemopoietic stem cell
disorders characterized by:
o Peripheral cytopenia
o Qualitative abnormalities of erythroid, myeloid and magakaryocyte maturation
o An increased risk to transform to acute leukaemia.
The overall incidence of MDS is 3.8 per 100 000 with 75% of cases being over the age of 65 years.
The diagnosis of MDS is dependent on careful examination of the peripheral blood film, bone marrow (BM)
and cytogenetics. MDS is biologically very heterogeneous; the precise pathophysiology in many cases
remains unknown. The recent recognition of familial cases of MDS suggests that at least some types of
MDS are likely to have a significant genetic component.

Table 1: The current (WHO) classification of MDS

1. Refractory anaemia (RA)


a. with ringed sideroblasts (RARS)
b. without ringed sideroblasts
2. Refractory cytopenia with multilineage dysplasia (RCMD)
3. Refractory anaemia with excess of blasts (RAEB)
a. RAEB-I (BM blasts 5-10%)
b. RAEB-II (BM blasts 11-20%)
4. 5q- syndrome
5. Unclassified MDS:
a. hMDS (5%-15%),
b. MDS with fibrosis,
c. t-MDS,
d. childhood MDS

Clinically MDS patients present with features of the cytopenia (infection, bleeding, tiredness) and their
management will include supportive (blood products, antimicrobials, growth factors [EPO, G-CSF]) and
specific therapy.
The specific therapies include:
i. Biological modifiers (immunosuppressive therapy, azacytodine)
ii. Oral chemotherapy (hydroxyurea)
iii. Intensive chemotherapy/stem cell transplantation (SCT)
a. AML type regimes
b. Allo/VUD standard/reduced intensity
Allogenic stem cell transplantation and intensive chemotherapy can prolong survival but only a minority of
MDS patients can benefit.

Since MDS has been found to be very heterogeneous it has been useful to develop scoring systems to
predict prognosis and decide on treatment for individual patients. One such scoring system is given below.
The IPSS scoring system converts risk to median survival and progression to AML.

Table 2: International Prognostic Scoring System (IPPS)

Risk category Score value Median survival (years) AML evolution (years)

41
Low 0 5.7 9.4
Intermediate-1 0.5-1.0 3.5 3.3
Intermediate-2 1.5-2.0 1.2 1.1
High >2.5 0.4 0.2

The score value is based on the blast count, cytogenetics and the degree of cytopenia. In general the more
intensive forms of specific therapy (including SCT) are reserved for patients with high risk MDS (high score)
and who are relatively young.

• Aplastic anaemia (AA) is characterized by the inability of the bone marrow (BM) to produce an
adequate number of circulating blood cells.
o It is associated with significant mortality and affects an estimated 2-5 people per million per year.
o AA can occur at any age, but there is a peak incidence between 15-24 yrs and after the age of 60
yrs.
o The vast majority (~75%) of these cases are classified as idiopathic, and the primary pathology
remains unexplained.
o Occasionally (~15% of cases), a drug or infection can be identified that precipitates the aplasia,
although it is not clear why only some individuals are susceptible.
o In approximately 10% of patients the disease is constitutional/inherited, where the disease is familial
and/or presents with one or more other somatic abnormalities. AA thus represents a heterogeneous
group of disorders (Table 3 below).

Table 3: Classification of aplastic anaemia (bone marrow failure)

Idiopathic (~70%) Vast majority


Inherited (~10%) Dyskeratosis congenita
Fanconi anaemia
Shwachman-Diamond syndrome
Secondary (~10-15%) Radiation Predictable
Drugs Predictable Cytotoxic agents
Idiosyncratic Chloramphenicol
Non-steroidals
Viruses Idiosyncratic Hepatitis viruses
Immune SLE

o In all subtypes of AA the numbers of “haemopoietic stem cells” are reduced. (failure of BM to
produce blood cells)
o Stem cell problem (CD34, LTC-IC)
o Immune attack
! Increased T cell subsets (skewed TCR VB repertoire)
! HLA-DR2 (DR15) association
! Increased γ-IFN, TNF-α, Fas
o In many patients with idiopathic AA it is possible to demonstrate that there is an immune attack on
the haemopoietic system.
o There is a close link between AA, paroxysmal nocturnal haemoglobinuria (PNH) and leukaemia.
o The 10-year cumulative incidence rates for myelodysplastic syndrome (MDS) and leukaemia are ~
10% and ~ 6%, respectively. Such observations suggest that AA can be regarded as a pre-
leukaemic disorder.
The symptoms and signs of AA result from the reduced numbers of circulating blood cells:
(i) anaemia leads to increasing fatigue and breathlessness,
(ii) leukopaenia (deficiency of leucocytes) leads to an increased risk of infections and
(iii) a tendency to bruise and bleed easily due to lack of platelets. Typically patients present with easy
bruising, gum bleeding or nose bleeding.

42
The diagnosis of AA is made on tests on the blood and BM (the blood will show cytopaenia and the marrow
will be hypocellular). These tests also allow it to be classified into severe aplastic anaemia (SAA) and non-
severe aplastic anaemia (NSAA).
Those with SAA have BM cellularity of <25% and 2 out of 3 of the following peripheral blood features:
(i) reticulocytes <1% (<20 x 109/l),
(ii) neutrophils <0.5 x 109/l, and
(iii) platelets <20 x 109/l
(iv) bone marrow <20% cellularity.
Patients with very severe AA (VSAA) have
(i) a neutrophil count of <0.2 x 109/l,
(ii) platelets <20 x 109/l,
(iii) and reticulocytes <20 x 109/l.
A family history of AA, somatic abnormalities on clinical examination and the results of special
investigations may point towards constitutional bone marrow failure.

The treatment for patients with SAA can be divided into


o Supportive care (blood products – leucodepleted and CMV negative, antimicrobials) and
o Specific therapy (e.g. immunosuppressive therapy and stem cell transplantation).
! Idiopathic
! Immunosuppressive therapy (ALG, CSA, G-CSF)
• <16 yrs 69-88% 5yr survival
• > 16 yrs 79-88% 5yr survival
! Haemopoietic stem cell transplantation
• Sib donors
• <16yrs 82-90% 5yr survival
• >16yrs 67-74% 5yr survival
• Alternative donors; 20-40% 5yr survival
! Late complications following immunosuppressive therapy include
! Relapse of AA (35% over 15 yrs)
! Clonal haematological disorders
• Myelodysplasia
• Leukaemia
• 10-40 % over 10 yrs
! Solid tumours

The two constitutional syndromes that are frequently associated with generalised BM failure/aplastic
anaemia (AA) are Fanconi anaemia (FA) and dyskeratosis congenita (DC). These two syndromes are now
also two of the best characterized and are providing important insights into normal haemopoiesis and how it
might be disrupted in AA. In new patients presenting with AA it is important to recognise if they have an
underlying genetic disorder (such as FA), as this will influence the precise management.

43
STEM CELL TRANSPLANTATION

Learning Objectives

! Understand the nature of autologous and allogeneic stem cell transplantation.


! Understand the indications for stem cell transplantation

Allogeneic stem cell transplantation (allo-SCT)


Involves the replacement of the patient’s haemopoietic stem cells (HSC), normally found
within the bone marrow, by a source of normal HSC, usually derived from an HLA-identical
sibling or volunteer unrelated donor.

Indications
Either when the patient’s own bone marrow has failed, in which case the goal of treatment
prior to the transplant is to suppress the patient’s immune system, or where the marrow is
affected by disease, e.g. leukaemia, where therapy pre-transplant must both eradicate the
disease and suppress the immune system.

! Bone marrow failure syndromes include aplastic anaemia and some rare congenital
disorders such as Fanconi anaemi and Dyskeratosis Congenita.
! Malignant indications for allo-SCT include the acute and chronic leukaemias, lymphoma,
myeloma, myelodysplasia and some solid tumours.

Autologous stem cell transplantation (auto-SCT)


Refers to the use of the patient’s own HSC as the replacement tissue after high dose chemo-
and/or radiotherapy.

! One of the first limiting toxicities of chemo- and radiotherapy is bone marrow failure.
! Higher doses can be given if the patient’s HSC are removed prior to the onset of treatment,
cryopreserved and then thawed and re-infused after the administration of the high dose
therapy.

Indications
Auto-SCT is used in situations where either the bone marrow is not affected by the disease,
e.g. some lymphomas, solid tumours, or can be rendered free of disease by prior therapy,
e.g. acute leukaemia, or where allo-SCT is not possible because of the age of the patient or
the lack of a suitable donor, e.g. myeloma, chronic lymphocytic leukaemia.

! Understand the pathogenesis of the complications of stem cell transplantation

Complications of stem cell transplantation

1. Toxicity of chemo-radiotherapy

! The combination of drugs and/or radiotherapy used to prepare the patient for the infusion
of HSC is known as the ‘conditioning regimen’.
! In both auto- and allo-SCT the most frequently used cytotoxic drugs are the alkylating
agents, eg, melphalan, busulphan and cyclophosphamide.
! These agents have well characterised side effects:
Alopecia Pulmonary fibrosis
Haemorrhagic cystitis Gastro-intestinal problems of nausea
Vomiting Liver toxicity known as veno-occlusive disease
Diarrhoea and mucositis Gonadal failure
Cardiotoxicity

! Irradiation is more commonly used in allo-SCT and is administered as total body irradiation
(TBI).
! TBI can also cause gastro-intestinal disturbance, pulmonary damage, pancreatitis,
parotitis, gonadal failure and veno-occlusive disease.

44
! All these complications are more common in older patients and in those in whom organ
damage is already present.

2. Pancytopenia due to myeloablation

The goals of the conditioning regimen are two-fold:


1. To eradicate the underlying disease and
2. To immunosuppress the patient sufficiently to prevent rejection of the incoming donor
tissue.

! Within a week of the therapy the haemoglobin (Hb), white cell and platelet counts will fall.
! This renders the patient susceptible to the consequences of anaemia, i.e. fatigue, lethargy,
dyspnoea and heart failure, leucopenia, (bacterial, viral and fungal infection) and
thrombocytopenia (bleeding and bruising).

! At this time the patient will be nursed in a single room using a ‘reverse barrier’ system, to
prevent the introduction of exogenous organisms into the room.
! Visitors will pay particular attention to hygiene, wearing plastic aprons and hand-washing.
! The patient will receive prophylactic antibiotics to try to prevent sepsis due to endogenous
organisms.
! The immediate period of neutropenia is of the order of 7-28 days, depending on the
conditioning regimen and the nature of the transplant, i.e. autologous or allogeneic.
! However, the effect on the immune system is prolonged and patients remain susceptible to
viral and fungal infections for up to one year post transplant.

3. Graft versus host disease

! An immune response when donor cells recognise the patient as ‘foreign’


! In allo-SCT, the donor’s cells are immunocompetent and their T-lymphocytes are capable
of recognising the patient as ‘foreign’.
! An inflammatory reaction, known as graft versus host disease (GvHD) occurs in most
patients undergoing allo-SCT, but is variable in severity.
! Acute GvHD occurs within 100 days of transplant and can affect the skin, gastro-intestinal
tract and liver.
! In its mildest form, it does not require treatment and is self-limiting.
! In its most severe form, affecting about 10% of recipients of sibling grafts and 20% of
recipients of unrelated volunteer cells, it is uniformly fatal.

! The intermediate forms require treatment with additional immunosuppression which in turn
increases the risk of infection.
! After day 100, on-going and de novo GvHD are referred to as chronic GvHD.
! Chronic GvHD affects skin, mucosal membranes, lungs, liver, eyes, joints

! This condition is similar to an auto-immune process with many of the features of


scleroderma.
! If the condition is severe enough to require on-going immunosuppression, patients remain
at risk of death from infection.

! GVHD can be prevented by the removal of the donor’s T-lymphocytes prior to infusion.
! This is usually achieved by incubating the donor’s HSC with a T-cell specific monoclonal
antibody, such as alemtuzamab (Campath) (ex vivo T-cell depletion), or by administering
this antibody at the same time as the infusion of HSC (in vivo T-cell depletion).
! Although this strategy improves the outcome of transplant in the short-term, there is an
increased risk of disease recurrence.
! This observation led to the concept of the ‘graft versus tumour’ effect, in which it was
recognised that the mechanism of cure through transplant was due to both the intensity of
the conditioning regimen and an on-going eradication of residual disease by
immunocompetent donor cells.
! This was further confirmed by the ability of the infusion of additional donor lymphocytes
(DLI) to restore remission in patients who had relapsed with their leukaemia after allo-SCT.

45
The combination of the risks associated with the conditioning regimen, prolonged
immunosuppression and pancytopenia and GvHD, give a transplanted related mortality, i.e.
risk of death due to the transplant as opposed to the disease, of approximately

! 5% in auto-SCT,
! 20-30% in sibling allo-SCT
! 30-50% in unrelated allo-SCT

Unrelated allo-SCT is therefore one of the most dangerous elective procedures performed as
a therapeutic strategy, and care must be taken to select only those patients most likely to
survive the procedure and benefit long-term.

Mx

Corticosteroids FK506
Cyclosporin A Mycophenylate mofetil
Monoclonal antibodies Photopheresis
Total lymphoid irradiation

Prevention
Methotrexate CsA plus MTX
Corticosteroids FK506
Cyclosporin A T-cell depletion

4. Long term effects of SCT

Patients who have been treated by SCT require life-long follow-up, particularly those who
received allo-SCT.

Late sequelae of transplant include


! Gonadal failure (premature menopause, permanent infertility)
! Cataract formation (particularly after TBI)
! Endocrine deficiencies (thyroid dysfunction, growth retardation and pubertal delay in
children)
! Pulmonary disease (fibrosis and bronchiectasis)
! Cognitive impairment
! Psychological problems.

The management of the transplant patient is complex and requires a highly effective multi-
disciplinary team approach.

! Be aware of the outcome of SCT for the more frequent indications

Choosing a Donor
! Well matched for tissue type - also known as HLA type
! Ideally a sibling (one on four chance of matching with each sib)
! If not, a volunteer unrelated donor or minimally mismatched family member

What affects the outcome of the transplant?


!Age <20=0,20-40=1,>40=2
!Disease phase Early=0, int=1, late=2
!Gender of R/D Female into male = 1
!Time to BMT <1 yr = 0, >1 yr = 1
!Donor Sib = 0, VUD = 1

Engraftment

46
When bone marrow infused during a bone marrow transplant takes or is accepted by the
patient and begins producing blood cells.
! WCC > 1.0 x 109/l
! Neutrophils > 0.5 x 109/l
! Platelets > 20 x 109/l
! Platelets > 50 x 109/l

Risk Factors for Infection


! Neutropenia
! Break down of protective barriers
! Decreased antibody levels
! Depressed T-cell immune responses

Allogeneic SCT Infections


Aspergillosis - 10-15% of all deaths
CMV: the troll of transplantation
Risk Factors for CMV disease:
o The patient’s serological status
o The donor’s serological status
o The type of stem cell donor (sibling, unrelated, haplo)
o The type of transplant (allogeneic, autologous, reduced conditioning)
o The CMV viral load

Harnessing the Immune System


! Controlling GvHD with gene therapy
! Mini-transplant procedures
! Directing the immune response specifically at the disease cells
! Directing the immune response specifically at the infection

Suicide Genes and DLI


! Retroviral mediated transfer of HSV-TK into donor lymphocytes prior to use
! Activation of suicide gene with ganciclovir at onset of GVHD
! I.e. If GvHD kicks off suicide gene is activated and all donor lymphocytes are killed off.
! EEC supported (1996-2004) multi-centre study to demonstrate safety & efficacy

47
Haematology – OBSTETRIC HAEMATOLOGY

Learning Objectives
• Understand the importance of the normal changes in blood parameters that occur during
pregnancy and their role in complications of pregnancy.
• Be able to distinguish normal gestational changes from pathological changes
• Understand the role of maternal testing in preventing and anticipating fetal disorders.

Why pay special attention to the haematological changes in pregnancy?

1. it avoids misdiagnosis e.g.: ITP or protein S deficiency


2. it reminds you to anticipate problems e.g. thrombosis and haemolytic disease of the
newborn (HDN)

Red and white cells in pregnancy


• A mild degree of ‘anaemia’ is common during pregnancy
• Anaemia leads to IUGR and prematurity
• Despite the rise in red cell mass during the second and third trimesters (up to 130% of
non pregnant level), there is a greater rise in plasma volume (up to 150%).
• Iron deficiency is common due to the increase red cell mass and requirements of the
fetus (total approx. 800mg)
• Folate requirements are also increased
o growth and cell division
o approximately additional 200 mcg/day required
• WHO recommends 60mg Fe + 400 mcg folic acid
• A slight ‘macrocytosis’ (rise of 5-10fl) is a normal feature of pregnancy in the absence of
folate or B12 deficiency. A mild rise in neutrophils with some left shift may also be seen

So, the full blood count in pregnancy:


• Mild anaemia
o red cell mass rises (130%)
o plasma volume rises (150%)
• Macrocytosis
o normal
o folate or B12 deficiency
• Neutrophilia
• Thrombocytopenia

Thrombocytopenia in pregnancy

Gestational thrombocytopenia
• A fall to >80 x109/l is regarded as normal ‘gestational thrombocytopenia’
• The mechanism is partly dilutional and partly increased consumption and occurs mostly in
the third trimester
• It accounts for 75% of low platelet counts in pregnancy and is benign

Immune thrombocytopenic purpura (ITP)


• ITP accounts for 5% of pregnancy associated thrombocytopenia
• Indicated by preceding thrombocytopenia or early onset
• Intervene for bleeding or to achieve platelets >50 for delivery
• Intravenous immunoglobulin or steroids are first line therapy
• The child is unpredictably at risk of thrombocytopenia so check cord FBC at birth
• Neonatal thrombocytopenia usually responds to i/v immunoglobulin

Preeclampsia
• 50% get thrombocytopenia in proportion to severity
• Mechanism is thought to be accelerated clearance

48
• Associated with subclinical coagulation activation (incipient DIC – normal PT, APTT)

Microangiopathic syndromes
Microangiopathic haemolytic anaemia –
• deposition of platelets in small blood vessels
• thrombocytopenia
• fragmentation and destruction of RBCs within vasculature
• organ damage (kidney, CNS, placenta)

HELLP (haemolysis, elevated liver enzyme tests, low platelets)


• has some overlap with preeclampsia but there is higher fetal and maternal morbidity
• usually gets worse for 48hours after delivery

TTP (thrombotic thrombocytopenic purpura) and HUS (haemolytic uraemic syndrome)


• are both more common in pregnancy
• features and treatment are as for those occurring outside pregnancy and are not aided by
delivery
• earlier than HELLP, usually 2nd trimester

Coagulation in pregnancy

• Procoagulant factors tend to rise


Factor VIII and vWF Increase 3 – 5 fold during pregnancy.
• Protein C and antithrombin do
Factor VII Increases 3 fold
not change significantly.
Fibrinogen Increases 2 fold • Fibrinolysis is suppressed
during pregnancy.
Protein S Falls to half basal
• This is because tissue
Plasminogen plasminogen activator (tPA) falls
Increase 3 fold and plasminogen activator
Antiplasmin inhibitor PAI-1 rises and PAI-2 is
PAI-1 produced by the placenta.

• The risk of thrombosis is further increased by the gravid uterus impairing venous return
from the legs.
• The changes in pregnancy result in a net prothrombotic state.
• This is partly manifest as an increase in activated protein C resistance (APCR) and an
increase in measures of thrombin generation.
• Overall the risk of pregnancy associated thrombosis is 1 per 1000 <35 years and 2 per
1000>35 years.
• The risk is further increased by other factors such as: immobility, BMI >29, thrombophilic
traits, previous thromboembolism, Caesarean section.
• Largest cause of maternal mortality

Obstetric disasters such as


• amniotic fluid embolism
o “the most catastrophic event in modern obstetrics”
o 1 in 20,000 – 30,000 births
o sudden onset shivers, vomiting, shock
o 86% mortality
o presumed due to Tissue Factor in amniotic fluid
• abruption placentae
• retained dead fetus
• severe preeclampsia
• sepsis
are potent causes of disseminated intravascular coagulation (DIC).

49
Haemoglobinopathy screening in pregnancy
All women have a FBC and haemoglobin electrophoretic analysis at booking.

• HPLC
o will identify Hb variants e.g. S, C, E
o quantifies Hb A2 (>3.5% → β thal)
• FBC
o MCH<27 pg used to identify possible beta thalassaemia trait
o MCH<25 pg possible alpha thalassaemia trait.
• Risk of alpha zero is assessed on basis of ethnic group (mostly Chinese, SE Asia,
Greece & Turkey).

• Abnormalities are followed up by testing partner and/or referral to haematologist.


• The aim is to avoid thalassaemia major/intermedia or Hb Barts hydrops fetalis, Sickle Cell
disease, and compound heterozygous states causing symptomatic sickle syndromes
such as compound heterozygosity for haemoglobins S and C (SC).

Red cell antibody screening in pregnancy


This is covered fully under blood transfusion.

Thrombophilia and adverse pregnancy outcomes


• Traits associated with an increased risk of thrombosis (Factor V Leiden, lupus
anticoagulant) are associated with complications of pregnancy:
o fetal loss
o growth restriction
o abruption
o severe PET
• The mechanism in most cases seems to be thrombosis in the placental circulation and
anticoagulation has been shown to improved pregnancy outcomes.
• Aspirin and heparin given to women with Hx of obstetric problem and sometimes with
Thrombophilia

Haemolytic disease of the newborn: HDN


• All pregnant women have blood analysed for ABORh blood group and presence of red
cell antibodies
• Benefits
o rapid provision of matched blood
o detection of women at risk of HDN
o detection of isoantibodies
• NICE recommends all women receive anti D at 28 and 34 weeks
• Shown to reduce sensitisation to 0.3%

50
Haematology

Haemolytic Anaemias

! Normal red cell life span 120 days


! Haemolysis defined by an increased rate of red cell destruction with reduction in red
cell life span.
! May be predominantly:
o Intravascular - within circulation
o Extravascular - removal/destruction by
o Reticuloendothelial(RE) system

Haemolysis may or may not be associated with anaemia. In the first instance the body
attempts to compensate for the increased breakdown. The following may occur:
a) the rate of red cell production can increase
b) the bone marrow can expand
c) younger red cells (reticulocytes) are seen in the peripheral blood in increasing numbers.

Patients with haemolysis are at particular risk of:


1) Folate deficiency due to the high turnover of red blood cells; this may lead to an
acute exacerbation of anaemia as a result of megaloblastic erythropoiesis.
2) Risk of profound anaemia (an aplastic crisis) in association with parvovirus B19
infection. Parvovirus infects erythroblasts and temporarily halts red cell production. In normal
individuals with a red cell life span of 120 days this has a negligible effect on thehaemoglobin
level. In situations where the red cell life span is shortened, however, halting red cell
production, even transiently, leads to a fall in haemoglobin level.
3) Propensity to gallstones (cholelithiasis)
4) Increased risk of iron overload and osteoporosis

Causes of haemolysis
Several which are split into:
Inherited versus acquired causes or intravascular vs. extravascular haemolysis

51
Intravascular
! Plasmodium falciparum malaria (“Blackwater fever”)
! ABO mismatched blood transfusion
! Cold antibody haemolytic syndromes
! Drug induced haemolytic anaemias
! Microangiopathic haemolytic anaemias e.g. chronic disseminated intravascular
coagulation (DIC), haemolytic uraemic syndrome
! Glucose-6-phosphate-dehydrogenase (G6PD) deficiency
! Paroxysmal nocturnal haemoglobinuria

Extravascular
! Autoimmune
! Alloimmune
! Hereditary spherocytosis

Features of haemolysis
1. Clinical: pallor, jaundice, splenomegaly (extravascular haemolysis), increased incidence of
pigment gall stones, pigmenturia, family hx.
2. Laboratory:
! Haematology:
! Polychromasia seen on blood film reflecting reticulocytosis
! Reticulocyte count is high
! Anaemia may be present
! Biochemistry: red cell breakdown produces
! Increased serum bilirubin
! Increased urinary urobilinogen
! Increased faecal stercobilinogen
! Increased serum LDH

If intravascular haemolysis occurs, haemoglobin is released into the circulation and binds to
plasma haptoglobins. The complex which forms is removed by reticuloendothelial (RE) cells.
When plasma haemoglobin is present in excess of that
which can be bound by haptoglobin free haemoglobin enters the urine and is absorbed by
renal tubular cells. Under these circumstances there are the following laboratory findings:
! Haemoglobinaemia
! Haemoglobinuria – hb in urine
! Haptoglobins are low or absent
! Haemosiderinuria – iron salts in urine

52
There are a wide variety of conditions associated with haemolysis.

Hereditary spherocytosis
! Autosomal dominant condition characterised by spherocytic red blood cells on the
blood film and splenomegaly.
! Caused by genetic defects leading to abnormalities in the proteins which link the
spectrin skeleton on the internal surface of the red cell membrane to the external
phospholipid bilayer. As a result of this, membrane is progressively lost in the spleen
as the red cells circulate and they become spherocytic.

Clinical and laboratory features:


! In 75% cases there is a family history
! Jaundice
! Anaemia
! General clinical and laboratory features of haemolysis (see above)
Laboratory diagnosis:
! Spherocytes are more sensitive to osmotic lysis than normal red cells and this may be
identified by the osmotic fragility test.
Management:
! Folate supplementation
! Splenectomy corrects the anaemia but is associated with risk of infection with
capsulated bacteria e.g. pneumococcus

Glucose–6-phosphate dehydrogenase (G6PD) deficiency


! Affects up to 400 million worldwide and prevalent in areas of malarial endemicity
! X-linked - clinical effects seen predominantly in hemizygous males and homozygous
females
! Enzyme catalyses first step in pentose phosphate(hexose monophosphate) pathway -
generates NADPH required to maintain intracellular glutathione(GSH) to protect cells
against oxidative stress.
! In most cases of G6PD deficiency under normal circumstances there is sufficient
residual activity to prevent haemolysis. However, NADPH generated will be insufficient
to protect a cell from additional oxidative stress and haemolysis will occur.

Clinical effects:
! Neonatal jaundice
! Acute haemolysis(triggered by oxidants/infection)
! Chronic haemolytic anaemia
! Between crises, the blood count is normal.
! Occasionally patients have a variant of the enzyme which leads to chronic
intravascular haemolysis, but this is rare

Three major types of triggers:


! Fava beans (broad beans)
! Infections
! Drugs; antimalarials (primaquine, pamaquin), antibacterial agents (sulphonamides,
ciprofloxacin and related drugs), Miscellaneous (dapsone, probenecid).

Management.
Infection should be treated, and any relevant drugs stopped. A high urine output should be
maintained and transfusion given if necessary.

53
Haematology

Multiple Myeloma

Summary
• a malignant disease of the plasma cells in the bone marrow.
• Proliferation of one clone of plasma cells (monoclonal proliferation) results in production of a
monoclonal immunoglobulin (Ig) molecule which can be detected in the serum or the urine.
• frequently associated with bone pain, anaemia, and renal failure.
• accounts for about 1% of all cancers; 3,000 new cases of myeloma each year in UK.
• incidence increases with age; most patients are over the age of 60 years
• the cause is unknown in most patients; radiation exposure is known to increase the risk.
• incurable in most patients, average survival 3-5 years.

Monoclonal Immunoglobulins
• there are normally many different plasma cell clones, producing many different Ig molecules
• In response to infection or inflammation, there is proliferation of a number of different
plasma
cell clones leading to an increased number of plasma cells in the marrow (a reactive
increase) and a polyclonal increase in Igs, which appear as a broad band in the gamma
region on serum electrophoresis.
• in myeloma one clone overgrows and produces one particular immunoglobulin molecule - a
monoclonal immunoglobulin, also termed a monoclonal protein (M-protein) or paraprotein -
which appears as a dense narrow band on electrophoresis. There is a reduction in normal
polyclonal Igs.
• the abnormal plasma cells may also produce free light chains, which are small enough to
pass into the urine (Bence-Jones protein or BJP). Individual patients may have plasma cells
which secrete a whole immunoglobulin alone, BJP alone, or both.

ESR in Myeloma
• a high level of Igs in the blood causes a raised ESR (erythrocyte sedimentation rate)
• the ESR is therefore raised in most patients with myeloma, but is also raised in conditions
where there is a polyclonal increase in Igs (infection and inflammation)
• in patients with myeloma who produce only free light chains and no serum paraprotein, the
ESR is normal

Clinical features
Bone pain & lytic lesions

Anaemia
Hyperviscosity syndrome

Recurrent Infections

Bleeding tendency

Renal Failure
Amyloidosis

Clinical Features of Myeloma


• Bone pain. This affects about 60% of patients. X-rays may show lytic lesions (punched out
holes). Generalised osteoporosis (thinning of the bone texture) is also common and there
may be compression fractures of the vertebrae, leading to back pain and occasionally to

54
compression of the spinal cord and neurological symptoms
• Hypercalcaemia - resulting from bone destruction. Causes dehydration, drowsiness,
confusion, constipation and renal damage.
• Renal failure. About 30% of patients have some degree of renal impairment and about 5%
have severe renal failure. This is most commonly due to BJP, which damages the tubules.
Other factors which can contribute include hypercalcaemia, infection, dehydration and drugs.
• Anaemia. This is common in myeloma patients.
• Increased susceptibility to infection, both bacterial and viral.
• Amyloid. About 10% of patients with myeloma develop light chain amyloidosis (AL amyloid),
in which the light chains are deposited in the tissues in the form of amyloid, causes
enlargement and stiffness of the tissue. (Amyloid=starch-like) The kidney is usually affected,
with glomerular damage leading to loss of albumen, low serum albumen level and
consequent oedema (nephrotic syndrome)
• Asymptomatic patients may be picked up by the finding of a raised ESR or abnormal protein
electrophoresis.

Diagnosis
• Confirmation of the diagnosis at least 2 of the following 3 features must be present:
• a paraprotein in serum or in urine
• >10% plasma cells in the bone marrow
• lytic lesions on X-ray.

Staging of multiple myeloma (Durie-Salmon)


• Stage I: all of the following
– Hb > 10g/dl
– Serum calcium value normal
– No lytic lesions or solitary plasmacytoma
– IgG<50g/l IgA<30g/l
– Urine light chain M-component < 4g/24hrs
• Stage II: fitting neither Stage I nor Stage III
• Stage III: one or more of the following
– Hb <8.5g/dl
– Serum calcium>3mmol/l
– Multiple lytic lesions
– IgG>70g/l IgA>50g/l
– Urine light chain M-component >12g/24hrs

Treatment - General Aspects


• Renal failure: correct dehydration, treat hypercalcaemia, dialysis if required
• Hypercalcaemia: rehydration, intravenous infusion of a bisphosphonate, e.g. pamidronate.
• Bone disease: local radiotherapy for pain, long-term bisphosphonate.
• Infection: treat promptly and vigorously, influenza vaccine annually.
• Anaemia: usually improves when the disease responds. Give EPO. Blood transfusions may
be needed

Specific Treatment
• Chemotherapy is the mainstay of treatment in myeloma. Oral or intravenous drugs may be
used, often combined with steroids. eg VAD (vincristine, adriamycin, dexamethasone
(preserves BM function)), melphalan (alkylating agent – can wipe out BM function) and
prednisolone.
! Maintenance interferon αprolongs survival after chemotherapy

• Thalidomide, analogues of thalidomide and bortezomib are being investigated.


• Radiotherapy for local areas of disease, e.g. for pain or cord compression
• High-dose therapy with stem cell support in younger patients (autologous or allogeneic)

Outlook with Treatment


• The average survival of patients treated with conventional chemotherapy is around 3 years,
although some patients may survive much longer

55
• A number of factors predict prognosis, of which most important is the serum beta-2
microglobulin level.
• Others include renal function, serum albumin, calcium and haemoglobin
• High dose therapy appears to prolong survival but is not definitely able to cure the disease.

Conclusions
• Myeloma remains incurable in the majority of patients
• Allogeneic SCT should be considered in pts <50yrs with a matched sibling
• Autologous SCT is the treatment of choice for most patients
• Post-transplant strategies such as RIC SCT, vaccination and ex-vivo T cell manipulation
require further evaluation
• Thalidomide has anti-myeloma activity but the precise role needs further evaluation
• Many new agents including IMiDs,PS-341, tyrosine kinase inhibitors, RANK-Fc and
monoclonal antibody treatment require further evaluation
• Participation in clinical trials should be encouraged

56
Haematology

Paediatric Haematology

Explain how healthy children differ haematologically from adults


Children differ biologically from adults so that normal ranges for haematological and other
variables differ from those of adults. They also differ between children of different ages. In
addition, there is no gender difference in haematological variables, such as the haemoglobin
concentration (Hb), in pre-pubertal children.

Explain how and why haematological conditions seen in children differ from those of
adults.
The range of haematological (and other) diseases seen in children differs from that
seen in adults. This is partly because microorganisms are often encountered for the first time
in childhood and because children’s immune responses to microorganisms and other
antigenic stimuli differ from those of adults. Systemic response is also different as children are
more likely than adults to respond to infections with lymphocytosis; ‘reactive’ lymphocytes are
also common because of the frequent encounters with new microbial antigens.

The range of neoplastic diseases is very different so that the exposure to aetiological factors
or the body’s response to such factors or both must differ. However, the range of other
diseases, (e.g. neoplasms) also differs. Some neoplasms are much more common in
children, e.g. acute lymphoblastic leukaemia while others are unknown or virtually unknown,
e.g. multiple myeloma.

Some childhood diseases, even certain leukaemias, have their origin in intrauterine
life so that we also have to consider that the exposure and response of the fetus to
potentially harmful intrauterine influences (e.g. maternal irradiation or intake of oncogenic
substances by the mother) also influences the range of diseases seen in children.
Many inherited conditions first present and have to be recognized and managed in
childhood. This is true of thalassaemia major, most cases of sickle cell anaemia and the
more severe variants of inherited coagulation defects and inherited haemolytic anaemias.
The interaction between the child and the environment may lead to the initial clinical
presentation of an inherited condition. Thus an inherited bleeding disorder may first be
diagnosed when a baby boy is circumcised or when he is learning to walk and starts falling
over.

Similarly, the first acute haemolytic crisis in a child (usually a boy) with glucose-6-
phosphate dehydrogenase (G6PD) deficiency may occur as a result of infection, exposure to
an oxidant drug or ingestion of fava beans; it may even occur in the neonate when the breast-
feeding mother ingests fava beans or when the baby is dressed in christening robes newly
removed from mothballs (containing naphthalene).
The same disease can have clinicopathological features in children that differ from
those seen later in life. This applies both to inherited conditions, e.g. sickle cell anaemia, and
acquired conditions, e.g. autoimmune thrombocytopenic purpura.
The child’s response to illness differs from that of an adult and children may also
differ from adults as to how they metabolise drugs. In addition, there may be only limited
information on drug safety in children. In children with serious illnesses, is often necessary to
use drugs that are not licensed for use in children. Growth retardation is a problem unique to
children. It can result from the direct effects of prolonged severe illness on growth, from the
damaging effects of therapy (e.g. irradiation of the spinal column) or from failure of puberty to
occur (e.g. in iron-overloaded children with thalassaemia major who have gonadal failure).

In treating children with serious conditions it is necessary to think of the very long-term
effects of treatment, e.g. lifelong hyposplenism if the spleen has to be removed, gonadal
failure and resultant infertility following chemotherapy, damage to the brain following cranial
irradiation, second malignancies following chemotherapy or radiotherapy (such as therapy
related acute myeloid leukaemia and an increased incidence of brain tumours following
cranial irradiation).

57
Describe haematological conditions seen in childhood that are either common,
important to diagnose or both.

Sickle cell disease (SCD)


! A generic term that covers all conditions leading to sickling of red blood cells with
resultant clinicopathological effects; including not only sickle cells anaemia (SS) but
also compound heterozygous states such as sickle cell/haemoglobin C disease (SC),
sickle cell/beta thalassaemia and other less common compound heterozygous
conditions.
! Sickle cell trait (AS) is NOT classified as sickle cell disease.
! Precise diagnosis (by blood count and film, haemoglobin electrophoresis (or High
Performance Liquid Chromatography—HPLC) and family studies) is essential, to
permit genetic counselling.
! Beta globin chain defect it does not present at birth but can present during the first six
months of life as haemoglobin F synthesis decreases and haemoglobin S synthesis
increases.
! ‘Hand-foot syndrome’ often initial presentation. Another presentation almost confined
to childhood is splenic sequestration and parents or other carers
! need to be taught how to recognize this.
! Sickle cell anaemia in the infant and child differs from the same disease in the adult
due to the distribution of red bone marrow (susceptible to infarction) differs—the
hand/foot syndrome
! Splenic atrophy/fibrosis develops in the first few years of life as a result of recurrent
sickling and infarction in the spleen. This removes the risk of splenic sequestration but
puts infants with sickle cell disease at high risk of fatal pneumococcal infection. Early
vaccination and regular penicillin therapy are essential and for this reason it is
important that sickle cells disease is diagnosed at birth rather than waiting for the first
clinical features to develop.
! Acute red cell aplasia as a result of infection by parvovirus B19 is a complication of
sickle cell disease that occurs mainly in children and adolescents. Once the primary
infection has occurred and an immune response has been mounted the condition does
not recur. In older children, manifestations of sickling are generally similar to those in
adults. However stroke is a particularly important complication of SCD in children,
although it is not confined to this age period.
! Folic acid is more important in a child with SCD as hyperplastic erythropoiesis requires
folic acid, gowth spurts require folic acid and the red cell life span is shorter so
anaemia can rapidly worsen.

Vaso-occlusion in the First Decade


Hand-foot syndrome
34% Acute chest syndrome 15%

Painful crises, about 40% per year

Stroke, cumulative incidence 4-8%

1 2 3 4 5 6 7 8 9 10
Age in Years

58
Thalassaemias
Genetic disorders of globin chain synthesis may be referred to as thalassaemias (reduced
synthesis) or haemoglobinopathies (synthesis of a structurally abnormal molecule)

Haemoglobin Globin chains Period when mainly present

A α2β2 Late fetus, infant, child and adult

A2 α2δ2 Infant, child and adult

F α2γ2 Fetus and infant

Beta thalassaemia major


! Condition manifests in the first 6 months of life as haemoglobin F synthesis decreases
but haemoglobin A synthesis does not occur or occurs at a greatly reduced rate
(however, it may be suspected at birth by guthrie spots..
! Precise diagnosis (by blood count and film, haemoglobin electrophoresis, family
studies and molecular analysis) is essential, to permit genetic counselling.
! Presentation is usually with pallor and failure to thrive.
! Treatment by blood transfusion and, in the older child, when iron overload is
developing, by iron chelation. This is usually by subcutaneous infusion of
deferoxamine (previously known as desferrioxamine) on 5 or 6 nights a week.
! Clinical effects of poorly treated thalassaemia major
o Anaemia, heart failure, growth retardation
o Erythropoietic drive bone expansion, hepatomegaly, splenomegaly
o Iron overload, heart failure, gonadal failure

Haemolytic Anaemia
! Haemolytic anaemia in a child can be inherited or acquired
! Severe forms of inherited haemolytic anaemias are usually manifest in childhood, even
in the neonatal period but children can also get most types of acquired haemolytic
anaemia
! Most cases of congenital haemolytic anaemia are inherited but not all
! Transplacental passage of antibodies can cause haemolytic disease of the newborn.
This is most often due to ABO or Rh antibodies
! Inherited haemolytic anaemias can be due to defects in
o Red cell membrane - Hereditary spherocytosis, Hereditary elliptocytosis
o Haemoglobin - SCD
o Red cell enzymes—glycolytic pathway - Pyruvate kinase deficiency
o Red cell enzymes—pentose shunt - G6PD deficiency
o Other rare conditions

Principles of diagnosis
!Is there anaemia?
!Is there evidence of increased red cell breakdown, e.g. jaundice, splenomegaly,
increased unconjugated bilirubin?

59
! Is there evidence of increased red cell production, e.g. increased reticulocyte count,
bone expansion?
! Are there abnormal red cells?

Glucose-6-phosphate dehydrogenase (G6PD) deficiency


! X-linked disorder is the most common type of haemolytic anaemia due to an intrinsic
red cell enzyme defect. Males who inherit an abnormal gene are invariably affected.
Heterozygote females usually have approximately 50% G6PD enzyme activity; the
random Lyonisation of X chromosomes means that rarely carrier females may be
severely affected.
! Presentation is usually with an acute episode of intravascular haemolysis on exposure
to certain drugs, infection or acute illness
! G6PD deficiency should be suspected in cases of prolonged neonatal jaundice or
when a baby, infant or child develops sudden pallor and jaundice and the blood count
and film shows anaemia and the presence of irregularly contracted cells.
! Most frequently seen around the Mediterranean Sea (e.g. in Greece, Italy and the
Middle East) and in those of African ancestry including Afro-Caribbeans.
! Diagnosis by G6PD assay but it should be noted that, in the common variant found in
Africans and Afro-Caribbeans, the level of G6PD is normal in reticulocytes and if there
is a reticulocyte response to acute haemolysis the assay may be normal. The blood
film is then critical in diagnosis and should lead to the assay being repeated when the
acute haemolytic episode is over.
! Other family members, particularly male siblings, should be investigated and
! appropriate verbal and written information should be given to avoid, as far as possible,
future haemolytic episodes.

Other inherited haemolytic anaemias


! Hereditary spherocytosis can present in childhood with pallor and jaundice or with
sudden onset of clinically apparent anaemia as a result of a parvovirus B19 infection.
Diagnosis is by means of the blood count and film, reticulocyte count, bilirubin
measurement and family studies.
! A direct antiglobulin (Coombs) test is sometimes necessary to exclude autoimmune
! haemolytic anaemia, which is another cause of spherocytosis.
! Other uncommon and rare causes of inherited haemolytic anaemia need to be
! recognized in this age range but diagnosis generally requires referral of children with
! unexplained anaemia, particularly if associated with jaundice, to a haematologist.

Haemolytic uraemic syndrome


! Most common acquired haemolytic anaemia observed in childhood usually as a result
of infection by a pathogenic E.coli that secretes verocytotoxin. This toxin damages
endothelial cells leading to impaired renal function and a microangiopathic haemolytic
anaemia.
! Commonest cause of ARFin children in the UK. Diagnosis is important so that the renal
failure, which is usually temporary, can be appropriately managed.
! A blood count, blood film and reticulocyte count are very important in diagnosis.
! The haemolysis is what is called a microangiopathic haemolytic anaemia which means
that the red cells are damaged in capillaries and are fragmented by the process; small
angular fragments and microspherocytes are formed

Inherited coagulation defects in children


! Children with severe inherited coagulation defects usually present with abnormal
bleeding either shortly after birth or in the first few years of life.
! Haemarthroses and bleeding into deep tissues are particularly characteristic.
! The differential diagnosis is with a battered child, with other causes of swollen joints
and with acquired causes of a bleeding disorder (e.g. acute leukaemia or autoimmune
thrombocytopenic purpura) and Henoch-Schönlein purpura.
! A high index of suspicion should lead to consideration of ethnic origin, a family history
and a coagulation screen and platelet count and then to further investigations,
depending on the results of initial tests.

60
! Acquired coagulation defects, e.g. disseminated intravascular coagulation in
meningococcal sepsis, do not usually enter into the differential diagnosis since the
clinical presentation is very different.

Autoimmune thrombocytopenic purpura (AITP)


! Previously known as idiopathic thrombocytopenic purpura
! In children it is usually an acute self-limiting condition whereas in adults it is much
more likely to be chronic and relapsing.
! Presentation is with petechiae and bruises. Sometimes there are also ‘blood blisters’ in
the mouth.
! The differential diagnosis is with non-accidental injury and with other causes of
thrombocytopenia, particularly acute leukaemia.
! A blood count and film are of critical importance. If the Hb and white cell count (WBC)
are normal and if no blast cells are seen on a careful examination of the film, a
diagnosis of acute leukaemia is highly unlikely.
! Often children are observed without treatment and in that case no bone marrow
examination is necessary. If the haemorrhagic manifestations are sufficiently severe,
treatment is indicated, either with corticosteroids or by infusion of high dose
immunoglobulin.
! If corticosteroids are to be given paediatricians often request a bone marrow
examination, to exclude a diagnosis of leukaemia.
! Immune thrombocytopenic purpura in children may be triggered by infection. Apart
from the clinical history, infection-related immune thrombocytopenic purpura does not
differ much from autoimmune thrombocytopenic purpura and the management is the
same. Because of the sudden clinical onset, ITP is not usually confused with inherited
thrombocytopenias.
! However if the observation of a low platelet count does not result from presentation
with bleeding but is incidental to investigation of another condition then both congenital
and acquired causes require consideration.

Inherited causes of thrombocytopenia or impaired platelet function.


Congenital thrombocytopenia and congenital conditions with impaired platelet function are
rare. Diagnosis requires family history, blood film and platelet count. The presence of
bleeding typical of a defect of platelet function or number, e.g. petechiae, bruises and
mucosal bleeding, in a child with a normal platelet count is an indication for platelet function
tests and/or a bleeding time. Because these conditions are rare, heterogeneous and difficult
to diagnose, referral to a haematologist is indicated when they are suspected.

Von Willebrand’s disease


Diagnosis of von Willebrand’s disease
o Family history (mainly autosomal dominant)
o Coagulation screen
o Factor VIII assay
o Bleeding time
o Platelet aggregation studies
Presentation of von Willebrand’s disease
o Mucosal bleeding
o Bruises
o Post-traumatic bleeding
Treatment of von Willebrand’s disease by giving Lower purity factor VIII concentrates

Acute leukaemia in children


! The leukaemia most typical of children is acute lymphoblastic leukaemia (ALL).
! Presentation is usually with pallor, bruising, lymphadenopathy and bone pain. In
children with enlargement of the thymus there may also be respiratory distress. On
examination, petechiae, hepatomegaly and splenomegaly may be found. The
differential diagnosis is with other causes of bleeding and anaemia.
! A blood count and careful examination of a blood film are critical in diagnosis.

61
! Blast cells typical of acute leukaemia have to be distinguished from immature
lymphocytes that often occur in children as a response to infection.
! A bone marrow examination is indicated in a child with unexplained anaemia and
thrombocytopenia even if no blast cells are detected in the blood film.
! ¾ can be cured. Current treatment is with complex protocols of oral, parenteral and
intrathecal chemotherapy and with supportive care to mitigate the effects of
! anaemia, thrombocytopenia, neutropenia and impaired immune responses.
! Early referral to a haematologist is mandatory if this diagnosis is suspected.

! In children <2yrs the most common type of acute leukaemia is acute myeloid
leukaemia (AML). AML also occurs in older children but over the age of 2 years it
comprises a much lower proportion of total cases of acute leukaemia.
! Clinical features similar to those of ALL—pallor, petechiae or bruising and fever. There
may be hepatosplenomegaly. Lymphadenopathy can occur in AML as well as ALL but
is less common whereas thymic enlargement does not occur.
! A blood count and blood film is usually adequate to confirm the diagnosis but
occasionally only a bone marrow aspiration reveals the increased blast cells.
! Unexplained pancytopenia is an indication for a bone marrow aspirate.
! As for ALL, early referral of suspected cases to a haematologist is mandatory.

Case History
• A 5-year-old boy of Indian ethnic origin presented with lymphadenopathy and a
mediastinal mass on chest radiology
• WBC 180 x 109/l, Hb 9.3 g/dl and platelet count 43 x 109/l
• What is the most likely diagnosis?
• What is the mediastinal mass?
Answers:
• The very high WBC (180 x 109/l) in a child means a diagnosis of leukaemia is almost
certain
• The low Hb (9.3 g/dl) and platelet count (43 x 109/l) are the result of bone marrow
infiltration
• The mediastinal mass is the thymus

Explain how healthy neonates differ haematologically from infants and older children
Neonates are haematologically even more different from adults than are infants and older
children. At birth the Hb is much higher, the WBC is higher and there may be nucleated red
cells in the circulation. The mean cell volume (MCV) is much higher. In addition to
haemoglobin A, there is still some haemoglobin F present and in premature babies this may
be a high percentage of total haemoglobin. Haemoglobin production is greatly reduced
immediately after birth so that Hb falls steadily to reach a lower level than is normal at any
other time of life around the end of the first year.

Explain why the range of haematological conditions seen in neonates differs from that
in infants and older children.
• All sorts of things can happen to a fetus that cannot happen outside the uterus. There
may be transplacental passage of damaging alloantibodies (e.g. anti RhD or
antiplatelet alloantibodies leading to alloimmune haemolytic anaemia or alloimmune
thrombocytopenia purpura). Maternal autoantibodies can also cross the placenta and
destroy fetal platelets.
• The fetus may bleed into the maternal circulation, leading to anaemia. If there are
identical twins with a shared placenta there may be bleeding from one twin to another
leading to anaemia in one and polycythaemia in the other.
• A fetus that is deprived of oxygen can also become polycythaemic. All these adverse
events affect the fetus but the effects are still present in the neonate. In addition to
the misadventures that may befall the fetus, the neonate may bleed during delivery,
e.g. from damage to a cord blood vessel.
• Neonates are also susceptible to infection and have a limited neutrophil reserve,
which impairs their response.
• The coagulations system of the neonate is also immature so that the normal ranges
differ from those at any other time of life.

62
• Neonates are also prone to vitamin K deficiency, which can lead to haemorrhagic
disease of the newborn.
• Prolonged jaundice in the neonate can have a haematological cause, e.g. haemolytic
disease of the newborn or G6PD deficiency.

Leukaemia can develop in utero and manifest in the neonate. Congenital leukaemia is
particularly common in Down’s syndrome. This specific type of neonatal leukaemia (also
sometimes called transient abnormal myelopoiesis or TAM) differs greatly from leukaemia in
older infants or children. The leukaemia is myeloid with major involvement of the
megakaryocyte lineage. The most remarkable feature is that it usually remits spontaneously
and relapse one to two years later occurs in only about a quarter of infants. There are
analogies with other childhood tumours, e.g. neuroblastoma

State the haematological conditions for which neonates are screened and explain why
such screening is done
• Neonates are screened for various biochemical abnormalities such as congenital
hypothyroidism and phenylketonuria. In the UK they are also now screened for sickle
cell disease and thalassaemia major. The same ‘Guthrie card’ is used for all these
purposes.This means that babies with sickle cell disease can be started on
prophylactic penicillin and mothers can be alerted to the need to check for splenic
sequestration.
• Babies with thalassaemia major can be started on regular blood transfusion once the
haemoglobin falls significantly.

63
Haematology

Haematology Quiz & EMQ

The student should be able to:


• develop a differential diagnosis when given a clinical photograph or a clinical history
and should be able to plan further relevant investigations
• recognize an abnormal laboratory result and be able to offer an interpretation and
explain what should be done next

Normal ranges we are expected to know:


• Have a reasonable idea of the normal ranges for WBC, Hb, MCV, platelet count in
adults
• If the WBC is abnormally high or abnormally low, should be able to work out whether
it is the count of neutrophils, lymphocytes or eosinophils that is causing the
abnormality in the total WBC
• Know that there are variations in haematological normal ranges related to gender,
age and ethnic origin

More details of the blood count

Percentages of white cells are meaningless unless you use them to produce an absolute
count
What are the two possible explanations of Neutrophils 1%, Lymphocytes 99% ?
Leukaemia?

64
FBC of an 83-year-old man with no abnormal physical findings

This was an incidental discovery, which has persisted. Likely diagnosis: CLL (smear cells give
you the clue)

FBC of a 58-year-old man

• RBC, HB and Hct are high


• Patient appears to be polycythaemic
– Is it a true or a pseudo-polycythaemia?
– Is he hypoxic from chronic lung disease or cyanotic heart disease?
– Does he have inappropriate erythropoietin secretion from a cyst or tumour
– Does he have an intrinsic bone marrow disease
• In this blood count there are no clues as to the cause of the polycythaemia. USS of
the liver, spleen and kidney should be carried out to exclude secondary causes.

FBC of a 67-year-old woman with facial plethora. She is a smoker and has coronary
artery disease.
• Blood results show that the RBC, Hb and PCV are high
• WBC, neutrophil count and basophil count are high. In severe infection WBC is
raised. Basophils not seen in sepsis but haematological neoplasms.
• Platelet count is high
• Most likely diagnosis is polycythaemia vera
Tests that might help to confirm this diagnosis might include:
• Bone marrow aspiration and trephine biopsy
• Serum erythropoietin
• You might also want to do other tests to exclude a secondary polycythaemia

65
Elderly man came to Haematology OPD

What do you suspect? AITP


On this day his platelet count was 13 x 109/l
A week earlier it had been 3 x 109/l with a normal Hb and WBC

What 3 possiblities could it be?


• Non-accidental injury
• Coagulation abnormality
• Thrombocytopenia
It could be ALL causing
thrombocytopenia or a coagulation
disorder such as Haemophilia A.

Case Report
• A 10-year-old girl presented with a painful right knee that had started when she
knocked her knee in a swimming pool
• Next day she had become unwell with malaise, anorexia and fever
• Her GP prescribed amoxicillin for ‘otitis media’
• Next day her mother took her to A+E
• She was afebrile with a right knee was painful and swollen. X-ray of the knee showed
patchy changes in density in the right medial tibial plateau.
Blood tests:
• WBC 6.6 x 109/l
• ESR 60 mm/hr (NR 0-10)
• C-reactive protein (CRP) 27 mg/dl (NR 0-1) – indicates sepsis
Diagnosis:
• Osteomyelitis resulting from Staphylococcus aureus infection
• Other diagnoses you might have considered are septic arthritis, haemorrhage into the
joint following minor trauma, non-accidental injury
• Fever is often present and the WBC is often increased in osteomyelitis but not
invariably
• There is a history of trauma in 35% so this should not deflect you from this diagnosis
• The CRP is normal in only 2% of cases

66
• Many children with a single swollen joint just have bruising and possibly an
effusion as the result of trauma
• However some have haemophilia or osteomyelitis or non-accidental injury

What is thrombotic thrombocytopenic purpura?


Diagnosed from a classic pentad of clinical features
• Microangiopathic haemolytic anaemia
• Thrombocytopenia
• Fever
• Neurological abnormalities
• Renal impairment
Pathogenesis:
! The underlying defect is a deficiency of a plasma protein called von Willebrand’s factor
cleaving protease (or ADAMST13)
! The deficiency is autoimmune in origin
! Low levels of the protease lead to high levels of very large multimers of vWF, which
cause platelet thrombi to form
! The widespread platelet thrombi are responsible for the clinicopathological features
! Treatment is by plasma exchange
! Platelet transfusions should NOT be given
! Although the disorder is immune in nature, corticosteroids are not the primary
treatment

Case Report
• A previously healthy 20-year-old man presented with fever, sore throat, malaise,
dyspnoea and abdominal pain
• Ten days before admission he had fallen on his left side and had attended Accident
and Emergency with pain in the left chest wall
• Chest X-ray had been normal but he was observed overnight
• On this presentation, temperature 39.70C, BP 115/95, pulse rate 96 beats/minute,
generalized lymphadenopathy, pharynx inflamed, mild hepatomegaly, spleen
palpable 2 cm below left costal margin, abdomen tender
• WBC 11.2 x 109/l, lymphocyte count 7.8 x 109/l, Hb 10.9 g/dl, numerous atypical
lymphocytes
• A screening test for infectious mononucleosis was positive. He had IgM antibodies to
EBV viral capsid antigen suggesting he had a recent EBV infection
Is there any cause for concern?
• Yes—he is anaemic, he has tachycardia and he has abdominal pain and tenderness?
What do you suspect and what do you do?
• Splenic damage and intraperitoneal haemorrhage should be suspected and urgent
imaging should be done
An urgent abdominal ultrasound showed splenomegaly and detachment of the
inferolateral part of the spleen, 9 cm from the tip with subcapsular haematoma formation.
There was no intraperitoneal bleeding. What should be done now?
• Emergency splenectomy was performed. Vaccinate for pneumococcus,
meningococcus and Haemophilus influenzae and prescribe life-long penicillin

Anaemia of chronic disease


Pathophysiology
• Reduction in red cell lifespan
• Cytokine release
– IFNg, IL1 and TNF
• Reduced proliferation of erythroid precursors
• Suppression of endogenous erythropoietin production
• Impaired iron utilisation
Rx:
• Treat underlying disease
• Recombinant erythropoietin
• Transfusion

67
Key points:
• Anaemia of chronic disease may be normochromic or normocytic
• Inflammatory markers are present
• Ferritin is high or normal
• Fe and transferrin are reduced
• The diagnosis is the causative condition is usually easy but can be
EXTRAORDINARILY DIFFICULT

Hodgkin’s Lymphoma
• 3 per 100 000 in UK
• Peak in adolescence and > 50 years
• Commonly presents with painless supra-diaphragmatic lymphadenopathy
• 1/3 present with B symptoms
– Fever, drenching night sweats or > 10% loss of body weight in last 6 months
• Epstein-Barr virus found in >79% of over 50s
Investigations:
• Expert histopathological review of biopsy
• ESR still useful for prognosis and monitoring
• CT/ MRI/ PET
Treatment depends on disease stage & bulk

Staging involved anatomical


stage (I, II, III, IV)
Absence or presence of B
symptoms (A or B)

Stage I and II
Radiotherapy+/-
chemotherapy

Stage III and IV


Combination chemotherapy
plus radiotherapy if bulky
mediastinal disease

68

Vous aimerez peut-être aussi